You are on page 1of 40

ST.

LOUIS REVIEW CENTER


LICENSURE EXAMINATION FOR TEACHERS (LET)

ANALYZING TEST ITEMS


Directions. Choose the letter of the best answer.
1. He holds the distinction of being the first Asian to receive the Nobel Prize for Literature
A. Wole Soyinka C. Po Chu-I
B. Yasunari Kawabata D. Rabindranath Tagore
The correct answer is D. Tagore won the Nobel Prize for Literature in 1913. Both (A) Soyinka and (B) Kawabata are
also Nobel Prize winners but these are not the correct answers; Kawabata won in 1968 and Soyinka 18 years later in
1986,. The Nobel Prize, which was first given in 1901, did not exist during (C) Po Chu-I’s time in 846 A.D.

2. Filial piety is a basic tenet of this school of thought


A. Taoism B. Confucianism C. Hinduism D. Buddhism
The correct answer is B. Respect and devotion to parents is important in establishing harmonious relationship and is a
fundamental concept of Confucianism. (A) Taoism emphasizes wu wei or non- action to achieve harmony with nature.
(C) Hinduism highlights the individual’s dharma or duty, while (D) Buddhism focuses on self-awareness to attain
enlightenment.

3. What does this excerpt from the Rigveda reveal about the Hindu belief on the god Purusha?

A thousand heads hath Purusha, a thousand eyes, a thousand feet.


One every side pervading earth he fills a space ten fingers wide.
The Purusha is all that yet hath been and all that is to be,
The Lord waxes greater still by food.
So mighty is his greatness: yea, greater than this is Purusha,
All creatures are one-fourth of him, three-fourths eternal life in heaven.

A. Purusha is the personification of natural phenomena.


B. The god Purusha is the source of mercy and salvation for all.
C. Purusha is the primal source of all creation.
D. Purusha is powerful and supreme over all earth.
The correct answer is C. Purusha is the Primal Person in a sacrificial ritual where his body parts are dismembered to
create the different parts of the world and the form caste system. (A) is incorrect because the deities such as, fire
(Agni), the sun (Surya), death (Yama), not Purusha, personify various natural phenomena. (B) is incorrect because the
excerpt does not describe Purusha as merciful savior; neither is (D) correct because the excerpt does not talk about
Purusha’s power and supremacy rather, Purusha as the Primal Sacrifice in the ritual.

4. What insight is suggested by this haiku from Bashō?

Poverty’s child –
He starts to grind the rice
And gazes at the moon.

A. Nature has a soothing effect on the human spirit.


B. Child labor is a reality in many Asian nations.
C. The poor dreams and are hopeful of better things in their life.
D. Life is a never-ending routine of work and leisure.
The correct answer is A. By contrasting the task of grinding rice with the boy’s observation of the moon, Bashō
evokes a sense of longing and captures the comforting effect of nature on the human soul. (B) Child labor is an actual
problem in many third world nations in Asia but it is not the subject of this haiku. (C) Gazing at the moon may be
suggestive of dreaming and hoping but this leaves out the essence that the imagery creates. (D) is incorrect because,
while the grinding of rice is routine, the haiku does not have the tone of boredom.

5. This excerpt from Soyinka’s Telephone Conversation indicates the universal issue of __________.

“ARE YOU DARK? OR VERY LIGHT?” Revelation came.


“You mean – like plain or milk chocolate?” Her assent was clinical, crushing its light
Impersonality. Rapidly, wave-length adjusted,
I chose, “West African sepia.” – and as an afterthought,
“Down in my passport.”
A. gender discrimination C. human rights violation
B. colonial mentality D. racial discrimination
The correct answer is D. The landlady’s questions about color indicate her deep-rooted prejudice against black
Africans even if they are educated and accomplished. (A) is incorrect because although the telephone conversation is
between a female house owner and a male caller the poem does not focus on gender discrimination. Neither is (B)
acceptable because the poem does not highlight colonial mentality. (C) may be considered because racial discrimination
is a form of human rights violation but this is not the best answer.

6. The Hindu belief that life is an illusion


A. dharma B. artha C. maya D. kama
7. Varnas, the theoretical basis for the caste system of the Hindus rests on __________
A. one’s birthright C. a person’s capacity and intelligence
B. karma of past thought and actions D. the quality of contribution to society
8. __________ is a collection of non-speculative hymns representative of the Aryan religious spirit often comparable to
the psalms in the Old Testament.
A. Bhagavad Gita B. Dhammapada C. Rigveda D. Upanishads

1
9. What rhetorical device is used in this line from the Ramayana?
“Dearly, loved, devoted Sita! daughter of a royal line.”
A. onomatopoeia B. alliteration C. assonance D. consonance
10. What do these thoughts of Dushyanta reveal about his character?
“A tranquil spot! Why should I thrill?/ Love cannot enter there – ”
A. indecisive B. emotional C. rational D. ambivalent
11. A collection of sayings raging from brief statements to more extended dialogues between Confucius and his students.
A. Analects B. Tao-Te Ching C. Book of Changes D. Manyusho
12. Known as the ‘old philosopher’, he is credited as the founder of Taoism
A. Lao-Tzu B. Confucious C. Chuang Tzu D. Li Po
13. The __________ is a Chinese poem sung to the tunes of popular melodies
A. shih B. li sao C. lu-shih D. tzu

Read the following excerpted dialogue from Injustice Done to Tou Ngo then answer questions 14 – 15

Donkey Chang: You’ve poisoned my father, Tou Ngo. Do you want to settle it in court or out of the court?
Tou Ngo: What do you mean ‘settle it in court or out of court?’
Donkey Chang: If you want to settle in court, I’ll take you there to be tried and cross-examined and put to the
torture. With a delicate body like yours you’ll find it hard to bear that. You’re bound to confess
to having poisoned my father. If you want to settle out of court, you’d better become my
wifejust as quick as you can. It’ll be doing you a favor.
Tou Ngo: I have not poisoned your father. I’ll go to court with you.

14. What does Donkey Chang’s attitude show of him as a man?


A. He has so much masculine pride C. He feels that he is the savior.
B. He belittles the strength of women. D. He regards women as objects.
15. What does line 9 reflect about Tou Ngo’s character?
A. She is picky with men. C. She is a very strong woman.
B. She is unafraid of suffering. D. She is truly innocent.
16. The __________ written by Sei Shōnagon records her irreverent reflections on life at the Imperial Court during the
Heian dynasty.
A. The Tale of Genji C. The Pillow Book
B. Things That Cannot Be Compared D. Essays in Idleness
17. This ethical concept suggests a sense of obligation or indebtedness which explains the sense of patriotism and
nationalism of the Japanese.
A. giri B. on C. seppuku D. kami
18. The earliest form of Japanese drama performed on an almost bare stage by a small but elaborately costumed cast of
actors wearing masks is called ___________
A. Nō B. Kabuki C. Jorori D. Kyogen
19. Atsumori compares warriors to images from nature in these excerpt from Motokiyo’s Atsumori. What do these
images reveal about his view of war?

Atsumori: . . . The leaves of the autumn of Juyei


Were tossed by the four winds;
Scattered, scattered (like leaves too) floated their ships.
And they, asleep on the heaving sea, not even in dreams
Went back to home.
Caged birds longing for the clouds –
Wild geese were they rather, whose ranks are broken
As they fly to southward on their doubtful journey. ..

A. War is a challenge for the brave who are unafraid of the risks.
B. War is hateful because warriors have no control over their lives.
C. War is the revenge of the strong and the powerful.
D. War is freedom for the dreams of the people.
20. What is the main idea of this haiku by Bashō?
So soon to die,
And no sign of it showing –
locust cry.
A. beauty of nature C. permanence of things
B. transitoriness of life D. reality of death
21. One of these features is not characteristic of African oral literature
A. repetition and parallel structure C. call-and-response
B. tonal alliteration D. repeat-and-vary
22. The __________ is an important kind of African moral tale intended for listeners to discuss and debate usually with
an open-ending.
A. Trickster B. Ashanti C. enigma D. origin
23. He is the leading figure of the Negritude movement.
A. Leopold Senghor B. Dennis Brutus C. Wole Soyinka D. David Diop
Read this excerpt from Diop’s Africa then answer questions 24-25.
Africa, my Africa,
Africa of proud warriors
In ancestral savannas,
Africa of whom my grandmother sings,
On the banks of the distant river
I have never known you.

2
24. Which description best describes the persona’s perception of Africa?
A. free and beautiful C. primitive and uncivilized
B. old and defeated D. mythical and elusive
25. What is the dominant tone of the poem?
A. regretful and frustrated C. sad and disdainful
B. angry and bitter D. invigorated and proud

ENHANCING TEST TAKING SKILLS


Directions. Choose the letter of the best answer.
1. This is the plot development of the Ramayana.
A. circular B. en medias res C. linear D. pyramid
2. A Sanskrit play like Shakuntala typically begins with __________
B. benediction upon the audience C. revelation of conflict
C. invocation of the muse D. flashback
3. The epic Ramayana is attributed to the poet __________
A. Vyasa B. Valmiki C. Kalidasa D. Bidpai
4. What is the theme of this excerpt from Tagore’s Gitanjali?
“Mother, it is no gain, they bondage of finery,
If it keeps one shut off from the healthful dust of the earth.”
A. The mother is a source of comfort and inspiration to her children.
B. Attachment to material wealth becomes a source of burden.
C. Nature conservation is important for one’s health and well-being.
D. A child’s affection to his/her mother shows tenderness.
5. Which emotion describes how the persona feels in this excerpt from The Taj Mahal?
“To you, my love, the Taj is a symbol of love. Fine.”
A. appreciative B. disbelieving C. supportive D. sarcastic
6. Fine arts and literature flourished during this dynasty which is viewed as the Golden Age of Chinese civilization.
A. Shang B. Ch’in C. Han D. T’ang
7. Chou-Shu-Jen is better known as __________ and is called the ‘father of the modern Chinese short story’
A. Mao Tse-Tung B. Lu Shun C. Li Ch’ing-chao D. Po Chu-I
8. This novel is based on the pilgrimage of the Buddhist monk Xuanzang to India in search of sacred texts.
A. On Learning to be an Indian C. The Tale of Genji
B. Record of a Journey to the West D. Dream of the Red Chamber
9. “To yield is to be preserved whole.” What figure of speech is used in this line from the Tao Te Cheng?
A. oxymoron B. paradox C. metonymy D. synecdoche
10. What does the falling of the plums in this excerpt from the Book of Songs mean?
Ripe plums are dropping, A. getting older
Now there are only seven. B. fewer men
May a fine lover come for me C. overripe fruits
Now while there is yet time. D. lost opportunities
Read Wang Wei’s poem below then answer questions 11-12
On an Autumn Evening in the Mountains
(trans. Arthur Christy)

How clear are the mountains after the new rain!


The dusk of the Autumn evening is pouring in,
As moonbeams filter through the pine trees.
Cool spring-water flows over white stones.
A lone washing-girl returns homeward by the bamboo grove.
The boatman sails his barge through the lotus patch.
Although Spring is long gone
Why cannot I linger over this pleasant view?

11. What is the dominant imagery in this poem?


A. visual B. auditory C. tactile D. olfactory
12. What emotion does the person express in the question at the end of the poem?
A. regret B. admiration C. nostalgia D. bitterness
13. All of these are novels of Yasunuri Kawabata except __________.
A. Confessions of a Mask C. Thousand Cranes
B. Snow Country D. Sound of the Mountain
14. This genre is one of the oldest and the most popular genre of expression and communication in the Japanese culture,
A. short story B. drama C. poetry D. essay
15. Ryunusuke Akutagawa’s In the Grove is distinct for its unique __________
A. characterization C. narrative technique
B. use of deus-ex-machina D. use of epiphany
16. The __________ is the best-preserved and the best-known African epic which is a blend of fact and legend of a
powerful leader who reestablished the Mandingo Empire of Old Mali.
A. Sundiata C. The Dausi
B. Askia the Great D. Monzon and the King of Kore
17. Chinua Achebe’s novel, the title of which is an epigraph from Yeat’s’ The Second Coming, is a lament over the
disintegration of Nigerian society represented by Okwonko, once a respected chieftain who looses his leadership with
the coming of the British colonizers.
A. Things Fall Apart B. No Longer at Ease C. A Man of the People D. Arrow of God
18. __________ is a South African novelist and short story writer whose major themes are on exile and alienation won
the Nobel Prize for Literature in 1991.
A. Bessie Head B. Barbara Kimenye C. Nadine Gordimer D. Ousmane Sembene
19. What is the theme of this African proverb?
“On the way to one’s beloved there are no hills.”
3
A. Real love knows no boundaries. C. Obstacles make a relationship strong.
B. Sacrifice is the gauge of true love. D. True love is tested by difficulties.
Read the excerpt below from Grace A. Ogot’s Rain
It was no longer a question of being the chief of hunger-stricken
people that weighed Labong’s hear. It was the life of his only daughter that
was at stake. At the time when Oganda came to meet him, he saw the
glittering chain shining around her waist. The prophecy was complete. “It
is Oganda, Oganda, my only daughter, who must die so young.” Labong’o
burst into tears before finishing the sentence. The chief must not weep.
Society had declared him the bravest of men. But Labong’o did not care any
more. He assumed the position of a simple father and wept bitterly. He loved
his people, the Luo, but what were the Luo for him without Oganda?

20. What is the main conflict of the story?


A. individual vs. self C. individual vs. society
B. individual vs. another D. individual vs. nature
21. One of these selections is a frame story.
A. Panchatantra C. The Tale of Genji
B. Record of a Journey to the West D. The Rain Came
22. This literary selection is an example of a satire.
A. R.K. Narayans’ The English Teacher C. Junichiro Tanizaki’s The Makioka Sisters
B. Cao Zhan’s Dream of the Red Chamber D. Mongo Beti’s The Poor Christ of Bombay
23. This novel is developed in a form of a recit, the French style of a diary-like confessional work.
A. The God of Small Things by Arundhati Roy C. The Pillow Book by Sei Shōnagon
B. The Houseboy by Ferdinand Oyono D. Song of Lawino by Okot P’Bitek
24. The indigenous literature of this country features the light entertainment of oral story telling.
A. India B. China C. Japan D. Africa
25. Many of this country’s writers committed suicide at the height of their careers.
A. India B. China C. Japan D. Africa

ANSWER KEY
ANALYZING TEST ITEMS ENHANCING TEST TAKING SKILL
1. D 10. C 19. B 1. B 10. A 19. A
2. B 11. A 20. B 2. B 11. A 20. C
3. C 12. A 21. B 3. B 12. B 21. A
4. A 13. D 22. C 4. B 13. A 22. D
5. D 14. C 23. A 5. D 14. C 23. C
6. C 15. D 24. D 6. D 15. C 24. D
7. D 16. C 25. D 7. B 16. A 25. C
8. C 17. B 8. B 17. A
9. B 18. A 9. B 18. C

ENGLISH & AMERICAN LITERATURE


Read the text and answer the questions by writing the letter of the best answer. Write your answers in the answer sheet.
1. What does the poem celebrate as shown in line 1-2?
My Heart Leaps Up
A. sadness in death C. familial bonding
William Wordsworth
B. reverence for nature D. sense of foreboding
2. What does the persona wish in the last two lines?
My heart leaps up when I behold
A. that he continues to be pious
A rainbow in the sky.
B. that he be a child once again
So was it when my life began;
C. that he continues to be connected to nature
So is it now I am a man;
D. that he fulfills his duties and responsibilities
So be it when I grow old, 5
3. What figure of speech does Wordsworth use in line 7?
Or let me die!
A. paradox B. metonymy C. oxymoron D. allusion
The Child is father of the Man;
And I could wish my days to be
ANSWER KEY Bound each to each by natural piety.
1. The best answer is B. “My heart leaps up…” connotes a strong love of or reverence for nature as represented by the
rainbow. It shows the persona’s extreme happiness, if not awe, with nature so he hopes to maintain natural piety
until his death. Options A, C, and D all focus on different subjects.
2. The best answer is C. The last two lines provide an apt conclusion to the strong reverence for nature presented in
lines1-2. In lines 3-6, the persona explains that he has been bound to nature since birth and he hopes to be until his
death. Options A and B both pick on key words piety and child to provide distracters, while option D proves a broad
option that does not appear in the text.
3. The correct answer is A. Line 7 is among the most famous paradoxes in literature. A paradox presents a seemingly
contradictory idea, but turns out to be true upon closer analysis. “The child is father of the man” connotes that much
wisdom can be learned from the innocence of the young; thus, the persona wishes to keep that youthful innocence
that connects the child to nature. Wordsworth himself reiterates this in Ode: Intimations of Immortality and in
Tintern Abbey.
Directions: Answer the questions by writing the letter of the best answer.
1. What is the tone of the following lines from Shakespeare’s Hamlet?

What a piece of work is a man! How noble in reason!

A. amazement B. mockery C. veneration D. sadness


2. The following lines from Robert Browning’s My Last Duchess exemplify what poetic strategy?
A. Aside
That's my last Duchess painted on the wall, B. Dialogue
Looking as if she were alive. I call C. Monologue
That piece a wonder, now: Frà Pandolf's hands D. Soliloquy
Worked busily a day, and there she stands. 4
Will't please you sit and look at her?
3. From what perspective is the following story told?
A. First person
"I could picture it. I have a rotten habit of picturing the bedroom scenes
B. Second person
of my friends. We went out to the Cafe Napolitain to have an aperitif and
C. Third person omniscient
watch the evening crowd on the Boulevard." from The Sun Also Rises
D. Third person limited
by Ernest Hemingway.

4. What type of irony does Shakespeare use in Anthony’s speech?

When that the poor have cried, Caesar hath wept: A. dramatic irony
Ambition should be made of sterner stuff: C. causal irony
Yet Brutus says he was ambitious; B. irony of situation
And Brutus is an honourable man. D. verbal irony

5. What do the following lines from William Blake exhort?

To see a World in a Grain of Sand A. to appreciate even the smallest of things


And a Heaven in a Wild Flower, B. to be extremely imaginative and creative
Hold Infinity in the palm of your hand C. to believe in fantasy like a child
And Eternity in an hour. D. to be strong and faithful to God

6. What poetic device is exemplified in the following lines from Edward


Taylor’s “Huswifery”?
A. irony of statement
Make me, O Lord, thy Spinning Wheel complete, B. pathetic fallacy
Thy Holy Word my Distaff make for me. C. a literary conceit
Make mine Affections thy Swift Flyers neat D. a paradoxical line
And make my Soul thy holy Spool to be.
My Conversation make to be thy Reel
And reel the yarn thereon spun of thy Wheel.
7. What does the persona in “Huswifery” ask God to do?
A. Complete him as a human being C. Mold him into what God wants him to be
B. Bless him with food and clothing D. Clothe him with the finest silk from God
8. Which two sound devices did Alexander Pope use in the following lines?

Soft is the strain when Zephyr gently blows, A. Assonance and consonance
And the smooth stream in smoother numbers flows; B. Alliteration and onomatopoeia
but when loud surges lash the sounding shore, C. Consonance and cacophony
The hoarse, rough verse should like the torrent roar: D. Onomatopoeia and assonance

9. What figure of speech is exemplified below?


“The wind stood up and gave a shout. He whistled on his two fingers.”
A. Allusion B. Metaphor C. Onomatopoeia D. Personification
10. What type of sonnet is exemplified in the following lines?

When I consider how my light is spent A. Elizabethan


Ere half my days, in this dark world and wide, B. English
And that one talent which is death to hide C. Petrarchan
Lodged with me useless, though my soul more bent D. Spenserian

11. Which statement best summarizes the Holy Sonnet X by John Donne?

And poppy or charms can make us sleep as well A. Death shall cease in the after life.
And better than thy stroke; why swell'st thou then? B. Death comes through poppy or charms.
One short sleep past, we wake eternally, C. Death takes so many forms and ways.
And death shall be no more; death, thou shalt die. D. Death should not be proud since it is not mighty.

12. What does the word “swell’st” in the Holy Sonnet X mean?
A. boast B. shrink C. grow D. swear
13. Which statement about love is true based on Shakespeare’s Sonnet 116?

Love's not Time's fool, though rosy lips and cheeks A. Love dissipates when lovers live apart.
Within his bending sickle's compass come: B. Love adapts to changing circumstances.
Love alters not with his brief hours and weeks, C. Love never wanes even in old age.
But bears it out even to the edge of doom. D. Love grows even to the edge of doom.

14. In “To the Virgins to Make Much of Time,” what is the persona’s main message?
A. Be wise in marriage to make life more worthwhile.
B. Marry now, or you may never have another chance.
C. Gather the rosebuds now, before the roses bloom.
D. Choose only lovers who, like roses, are of the highest order.
15. Which word best describes the speaker in “To Lucasta, on Going to the Wars”?

Tell me not, Sweet, I am unkind, A. cold-hearted


That from the nunnery
Of thy chaste breast and quiet mind 5
To war and arms I fly.
B. sweet-tongued
C. honorable
D. modest

16. To what sensory perception do the following lines from James Joyce’s Araby appeal?
A. auditory
“…we ran…to the dark dripping gardens to the back doors of the dark dripping
B. olfactory
gardens where odors arose from the ashpits, to the dark odorous stables
C. gustatory
where a coachman smoothed and combed the horse or shook music from the
D. tactile
buckled harness.”

17. What does the lamb in “The Lamb” symbolize?


A. Faith and loyalty
Little Lamb, who made thee?
B. Innocence and purity
Dost thou know who made thee?
C. Weakness and hopelessness
Gave thee life, and bid thee feed,
D. Helplessness and dependence
By the stream and o'er the mead;

18. Which of the following best states the theme of Ozymandias?

"My name is Ozymandias, king of kings: A. Power and arrogance are both destructive.
Look on my works, ye Mighty, and despair!" B. Temples and statues are witnesses to history.
Nothing beside remains. Round the decay C. Powerful rulers and great civilizations perish.
Of that colossal wreck, boundless and bare D. Life is short and time is fleeting.
The lone and level sands stretch far away.

19. What 17th Century philosophy does Browning assert in the following lines from Rabbi Ben Ezra?

Ay, note that Potter’s wheel, A. anagnorisis


That metaphor! and feel B. carpe diem
Why time spins fast, why passive lies our clay,— C. peripeteia
Thou, to whom fools propound, D. romanticism
When the wine makes its round,
“Since life fleets, all is change; the Past gone, seize to-day!”

20. What lesson does the speaker learn in A.E. Housman’s When I Was One-and-Twenty?
'The heart out of the bosom A. The speaker realizes the value of listening to pieces of advice.
Was never given in vain; B. The speaker learns the foolishness of disobeying his elders.
'Tis paid with sighs a plenty C. The speaker realizes the folly and pain of youthful love.
And sold for endless rue.' D. The speaker learns the stupidity of wasting his youth.
And I am two-and-twenty,
And oh, 'tis true, 'tis true.

ANSWER KEY
1. B 5. A 9. D 13. C 17. B
2. C 6. C 10. C 14. B 18. C
3. A 7. A 11. D 15. C 19. B
4. D 8. B 12. A 16. B 20. C

Directions: Answer the questions by writing the letter of the best answer.
1. How does Shelley regard the west wind in the following ode?
A. It is responsible for preserving life. From Ode to the West Wind
B. It can both wipe out and maintain life. Her clarion o'er the dreaming earth, and fill
C. It is a wild spirit in nature that is very strong. (Driving sweet buds like flocks to feed in air)
D. It is strong but weak since it is everywhere. With living hues and odours plain and hill:
Wild Spirit, which art moving everywhere;
Destroyer and Preserver; hear, oh, hear!
2. How does the speaker picture God in the following sermon?

The God that holds you over the pit of hell, much as one holds a spider; or some loathsome
insect, over the fire, abhors you, and is dreadfully provoked: his wrath towards you burns like
fire; he looks upon you as worthy of nothing else, but to be cast into the fire; he is of purer eyes
than to bear to have you in his sight; you are ten thousand times more abominable in his eyes,
than the most hateful venomous serpent is in ours.

A. incensed B. abominable C. assertive D. vengeful


3. Paradise Lost is considered among the greatest epics in English. Which of the following was the basis for this epic
poem?
A. treachery of Judas Iscariot C. fall from God’s grace
B. the passion of Christ D. sinning of Adam and Eve
4. What does the speaker mean in the following lines?
A. Let’s continue writing poetry to immortalize us.
“Let’s so persevere B. Let’s have faith in God and He will keep us alive.
That when we live no more, we may live ever” C. Let’s be true to our love, and we will be joined in eternity.
D. Let’s have lots of children to remember us when we die.
From To My Dear and Loving Husband

5. Which of the following is NOT an example of Gothic literature?

6
A. Dracula B. Lord of the Rings C. FrankensteinB. D. Tell Tale Heart
6. According to the speaker in Sanburg’s "Chicago," how would most others describe the city?
A. Admirable
They tell me you are wicked and I believe them, for I B. Amusing
have seen your painted women under the gas lamps C. Immoral
luring the farm boys. D. Vibrant

7. What does the speaker like about Chicago as shown in the following lines?
Come and show me another city with lifted head singing A. Its vitality
so proud to be alive and coarse and strong and cunning. B. Its wickedness
Flinging magnetic curses amid the toil of piling job on C. Its indifference
job, here is a tall bold slugger set vivid against the D. Its progress
little soft cities;
8. Who are the summer soldier and the sunshine patriot Paine alluded to in The Crisis?
A. The cowards who love their country less
THESE are the times that try men's souls. The summer
B. The brave men and women in the country
soldier and the sunshine patriot will, in this crisis,
C. The happy optimistic people
shrink from the service of their country; but he that
D. The former heroes of the revolution
stands it now, deserves the love and thanks of man
9. What does that the speaker lament over in the following lines?
A. Roses will always be roses despite their variety.
"What's in a name? That which we call a rose by any B. Their names keep Romeo and Juliet apart.
other name would smell as sweet". - (Romeo and C. Romeo and Juliet will always love one another.
Juliet Act II, Scene II) D. Changing names will help Romeo and Juliet.

10. Which of the following is an example of novel of the soil?


A. The Good Earth C. Catcher in the Rye
B. Bread and Wine D. Sound and the Fury
11. What does the speaker celebrate in “The Soul Selects her own Society”?
The soul selects her own A. conformity C. life and freedom
society, B. community D. self-imposed isolation
Then shuts the door;
On her divine majority
Obtrude no more.
12. What do the following lines reveal about the world?

"All the world’s a stage, and all the men and women merely players.
They have their exits and their entrances; And one man in his time plays
many parts" - (As You Like It, Act II, Scene VII)

A. Life is just like going to the theater. C. Life is but an empty, senseless dream.
B. People have different roles to play in life. D. People live and die at different times.
13. What truth about humans do the following lines from A Noiseless Patient Spider reveal?
A. People need food and shelter
And you, O my Soul, where you stand,
B. People search for their meaning
Surrounded, surrounded, in measureless oceans of space,
C. People need friends and families
Ceaselessly musing, venturing, throwing,--seeking
D. People endlessly seek to create
the spheres, to connect them;
Till the bridge you will need, be form'd--till the ductile anchor hold;
Till the gossamer thread you fling, catch somewhere, O my Soul.

2. Which of the following is the resounding theme of contemporary stories like Hemingway’s A Clean and Well
Lighted Place and Anderson’s Hands?
A. alienation from the society C. respect for the old
B. melancholia in solitude D. contentment in life
3. Who is alluded to as the Captain in the following lines from Whitman’s poem?
A. Abraham Lincoln
O`captain! My Captain! Our fearful trip is done, B. George Washington
The ship has weather’d every rack, the prize we sought is C. John F. Kennedy
won. D. Thomas Jefferson
4. In the passage, which of the following best describes the speaker's attitude toward the very rich?
Let me tell you about the very rich. They are different from you and me. They possess and enjoy early,
and it does something to them, makes them soft where we are hard, and cynical where we are trustful, in a
way that, unless you were born rich, it is very difficult to understand. They think, deep in their hearts, that they
are better than we are because we had to discover the compensations and refuges of life for ourselves.
Even when they enter deep into our world or sink below us, they still think that they are better than we are. They
are different.
A. He finds their pessimism alarming and unwarranted. C. He believes that the rich know more than others do.
B. He finds them so different from the rest of society D. He thinks that he understands their way of life.
5. What is the tone of the speaker in the previous passage?
A. Optimistic B. Laconic C. Pessimistic D. Sarcastic
6. What do the novels of Bronte, Eliot, Gaskell and Dickens reveal about fiction produced during the Victorian period in
English Literature?
A. They closely represent the real social life of the times.
B. The novels were long and full of psychological musings.
C. They concentrate on the effect of industrialization on cities.
D. They were largely produced by upper middle-class women.

7
7. What do the last two lines from Freneau’s The Wild Honeysuckle reveal about life?
From morning suns and evening dews A. Life is just an hour.
At first thy little being came; B. Life is frail.
If nothing once, you nothing lose, C. Life is short.
For when you die you are the same; D. It is like a flower.
The space between is but an hour,
The frail duration of flower.
8. What do the following lines from Wordsworth’s Psalm of Life reveal about heroes and heroism?

Lives of great men all remind us A. Anybody can be a hero.


We can make our lives sublime, B. Heroes are often forgotten.
And, departing, leave behind us C. Heroes are easy to find
Footprints on the sands of time; D. It is easy to do heroic acts.

ANSWER KEY
1. B 5. B 9. B 13. B 17. D
2. A 6. C 10. A 14. A 18. A
3. D 7. A 11. C 15. A 19. C
4. C 8. A 12. B 16. B 20. A

OLD ENGLISH LITERATURE


A. Anglo-Saxon Literature/Old English Literature (5 th Century-1066)
1. Old English literature survived in four manuscripts: Caedmon’s hymn, Vercelli Book, Exeter Book, and Nowell’s
Codex.
2. Poems are handed down from generation to generation through oral tradition. Scops or wandering bards recited
the poems with the accompaniment of the harp.
3. Anglo-Saxon literature include genres such as epic poetry, hagiography, sermons, Bible translations, legal works,
chronicles, riddles,
4. The verses are highly alliterative in nature with frequent pauses called caesura in each line. They also make
elaborate metaphors known as kennings. Kennings use compound poetic phrases to substitute for the name of a
person or thing.
5. Examples of kennings in Beowulf:
 Battle: the storm of swords
 Blood: fighting gear/battle gear-armor, battle-sweat
 Dragon: the twilight-spoiler
 Lord/King: the protector of warriors; ring-giver, dispenser of treasure
 Queen: peace-bringer among nations
 Sea: swan-road, the whale-road, the sea-bird's baths
 Ship: bent-necked wood, the ringed prow, the foamy-necked, the sea-wood, the sea-farer
 Sword: light of battle, the leaving of the file, battle-lightning
B. Representative Writers/Works
1. The Venerable Bede (673-735) was a monk at the Northumbrian monastery of Saint Peter at Monkwearmouth.
He was regarded as the greatest Anglo-Saxon scholar who wrote the Ecclesiastical History of the English People,
which earned for him the title Father of English History.
2. Alfred the Great (848?-899) was king of the southern Anglo-Saxon kingdom of Wessex from 871-899. Not only
did he defend the kingdom against Danish Vikings, but he also helped save and revive the Anglo-Saxon culture.
He wrote in his native tongue and encouraged scholarly translations from Latin into Old English (Anglo-Saxon).It
is believed that The Anglo-Saxon Chronicle was begun during his reign.
3. Cædmon (7th century) was an unlearned cowherd who was inspired by a vision and miraculously acquired the
gift of poetic song. His only known surviving work is Cædmon's Hymn, the nine-line alliterative vernacular praise
poem in honor of God.
Now let us praise the Guardian of the Kingdom of Heaven
the might of the Creator and the thought of his mind,
the work of the glorious Father, how He, the eternal Lord
established the beginning of every wonder.
For the sons of men, He, the Holy Creator
first made heaven as a roof, then the
Keeper of mankind, the eternal Lord
God Almighty afterwards made the middle world
the earth, for men.
- Caedmon, Hymn, St Petersburg Bede
4. Cynewulf (8th century) is regarded as one of the pre-eminent figures of Old English Christian poetry. His poems
include The Fates of the Apostles, Juliana, Elene, and Christ II or The Ascension.
5. Beowulf is the National epic of England and the most notable example of the earliest English poetry, which
blends Christianity and paganism.
 Beowulf appears in the Nowell codex manuscript from the 8th to the 11th century
 It tells of Beowulf's three fierce fights with the monster Grendel, the equally ferocious mother of Grendel,
and the fiery dragon. By conquering them, Beowulf saves his people from destruction.
6. Dream of the Rood is one of the earliest Christian poems preserved in the 10 th century Vercelli book. The poem
makes use of dream vision to narrate the death and resurrection of Christ from the perspective of the Cross or
Rood itself.
7. Heroic Old English poems: The Battle of Brunanburg and The Battle of Maldon
8. Old English Lyrics: The Wanderer and The Seafarer

MIDDLE ENGLISH PERIOD


A. Literature during the Middle English Period

8
1. Romances rank among the most popular literary forms of the Middle Ages. Sir Gawain and the Green Knight is an
example of a metrical romance, that is, a long rambling love story presenting knightly adventures and courtly love.
2. Religious dramas also became popular since they were performed as whole cycles of short plays in religious festivals.
 Miracle plays focused on real or fictitious accounts of the life, miracles, or martyrdom of saints. The genre
evolved from the liturgical dramas of the 10 th - 11th centuries, which were intended to enhance church calendar
festivals.
 Mystery plays represent a scene from the Old or New Testament. They are also known as pageants or as
Corpus Christi plays.
 Morality plays dramatized the typical content of a homily or a sermon and personified such abstractions as
Health, Death, or the Seven Deadly Sins and offered practical instruction in morality.
 Everyman is regarded as the best of the morality plays. From Everyman
It talks about Everyman facing Death. He summons the Everyman:
help of all his friends but only Good Deeds is able to help Methinketh, alas, that I must be gone,
him. Characters in this morality play are personifications of To make my reckoning and my debts pay,
abstractions like Everyman, Death, Fellowships, Cousins,
For I see my time is nigh spent away.
Kindred, Goods, Good Deeds, etc.
3. English and Scottish ballads preserved the local events, beliefs, Take example, all ye that do hear or see,
and characters in an easily remembered form. One familiar How they that I loved best do forsake me,
ballad is Sir Patrick Spens, which concerns Sir Patrick’s death Except my Good-Deeds that bideth truly.
by drowning.
Good-Deeds: All earthly things is but vanity:
B. Representative Writers
1. Pearl Poet (14th century) is generally remembered for his Beauty, Strength, and Discretion, do man forsake,
narrative poem Sir Gawain and the Green Knight - the best Foolish friends and kinsmen, that fair spake,
example of the romance of the Middle Ages. All fleeth save Good-Deeds, and that am I.
2. Geoffrey Chaucer (1340?-1400) was one of the world's greatest storytellers who is best remembered for his
frame narrative The Canterbury Tales.
3. Sir Thomas Mallory wrote Le Morte d'Arthur, a collection of stories about King Arthur and his Knights of the
Round Table culled from the Arthurian legends.

THE 16th CENTURY (THE RENAISSANCE)


A. Literature during the Renaissance
1. The Golden Age of English literature reigned from 1485 to 1660. William Caxton established movable printing in
England and published Mallory’s Le Morte d’Arthur. Soon after, there was a marvelous increase in the production
and quality of English literature.
2. Poets took up more secular views and writers wrote in praise of peace, of springtime, and of heavenly and earthly
love. The sonnet, a 14-line iambic pentameter poem, became the most favorite poetic form.
3. Humanism was the predominant philosophical thought in 16 th century England.
 Humanism encouraged the study of Greek and Roman classical literature. It reformed education by
making literary expression of paramount importance for the cultured person.
 Humanist writers glorified the individual and believed that the human being was the measure of all things
and had unlimited potential.
 Humanists promoted the concept of the well-rounded Renaissance man who was proficient in both
intellectual and physical endeavors.
B. Representative Writers
1. Christopher Marlowe (1564-1593) is considered as the Father of the English Tragedy best remembered for his
plays Tamburlaine and Doctor Faustus.
 His Dr. Faustus powerfully exemplifies the sum total of the Song – to Celia
intellectual aspirations of the Renaissance. In the play, Dr. Ben Jonson
Faustus sells his soul to the devil in exchange of power and
Drink to me, only with thine eyes,
knowledge.
And I will pledge with mine;
2. Edmund Spenser (1552-1599) left an unfinished work entitled The
Or leave a kiss but in the cup,
Faerie Queene which is an elaborate allegory built on the story of a
And I'll not look for wine.
12-day feast honoring the Queen of Fairyland (Queen Elizabeth I).
The thirst, that from the soul doth rise,
 Each verse in the Spenserian stanza contains nine lines: Doth ask a drink divine:
eight lines of iambic pentameter, with five feet, followed by a But might I of Jove's nectar sup,
single line of iambic hexameter, an "alexandrine," with six. The I would not change for thine.
rhyme scheme of these lines is ababbcbc-cdcdee.
3. Ben Jonson (1573-1637) was a poet, dramatist, and actor best I sent thee late a rosy wreath,
known for his satirical plays Volpone, The Alchemist, and Not so much honoring thee,
Bartholomew Fair. He also As giving it a hope, that there
Sonnet 18 wrote a good number of lyric poems.Some quotable quotes from Shakespeare
4. William Shakespeare is the great genius of the Elizabethan Age It could not wither'd be.
William Shakespeare
(1564-1616). He wrote more than 35 plays as well as1.154 "The But thou
play’s the thing wherein
sonnets I'll thereon
catch thedidst only breathe,
conscience of the
andShall
2 narrative poems –Venus and Adonis and The king".
Rape - (Hamlet
of Act II, SceneAnd
II) sent'st it back to me:
I compare thee to a summer's day?
Lucrece.
Thou art more lovely and more temperate. 2. "All the world’s a Since when
stage, and all it
thegrows,
men andand smells,
women I swear,
merely
Not of itself, but thee.
 Hiswinds
Rough sonnets,
do shakealsotheknown
darling as
budsthe players.are
Elizabethan sonnet,
of May, They have their exits and their entrances; And one man
Andcomposed
summer's oflease hath all too short in his time plays many parts" - (As You Like It, Act II, Scene VII)
three quatrains and aone
date.
heroic couplet with the
rhyme scheme - abab-cdcd-efef-gg. 3. "Good Night, Good night! Parting is such sweet sorrow, that I
Sometime too hot the eye of heaven shines, shall say good night till it be morrow." - (Romeo and Juliet, Act II,
And often is his gold complexion dimm'd; Scene II)
And every fair from fair sometime declines, 4. "What's in a name? That which we call a rose by any other name
By chance or nature's changing course untrimm'd; would smell as sweet". - (Romeo and Juliet Act II, Scene II)
5. "If you prick us, do we not bleed? if you tickle us, do we not
But thy eternal summer shall not fade laugh? if you poison us, do we not die? and if you wrong us, shall
Nor lose possession of that fair thou ow'st; we not revenge?". - (The Merchant of Venice, Act III, Scene I)
Nor shall Death brag thou wander'st in his shade, 6. "Cowards die many times before their deaths; The valiant never
When in eternal lines to time thou grow'st: taste of death but once." - (Julius Caesar, Act II, Scene II).
9
7. "How sharper than a serpent's tooth it is to have a thankless
So long as men can breathe or eyes can see, child!" - (King Lear, Act I, Scene IV).
So long lives this, and this gives life to thee.
5. The King James Bible is one of the supreme achievements of the English Renaissance. This translation was ordered
by James I and made by 47 scholars working in cooperation. It was published in 1611 and is known as the Authorized
Version. It is rightly regarded as the most influential book in the history of English civilization.

THE 17th CENTURY


A. Literature of the 17th Century
1. The 17th century is also called the Age of Transition or the Age of Revolution because a new middle class began
to show its power in this century. It also coincided with the Age of Exploration and scientific investigation.
2. It was also an Age of Prose. Interest in scientific detail and leisurely observation marked the prose of the time. This
new writing style emphasized clarity, directness, and economy of expression. It first appeared just before 1600 in the
Essays of Bacon.
3. Carpe diem was a recurrent theme of many lyrics in the 17th Century. Carpe diem, a Latin term which means “seize
the day” came from Horace’s Odes. The term reminds people of the transience of life and admonishes people to “eat
now and be merry for tomorrow we die.” The most celebrated examples in English literature are Marvell's To His
Coy Mistress and Herrick's To the Virgins, To Make Much of Time.
B. Representative Writers
1. Francis Bacon was a famous English essayist, lawyer, philosopher and statesman who had a major influence on the
philosophy of science. He is hailed as the Father of Inductive Reasoning and the Father of the English essay.
Philosophically, Bacon sought to purge the mind of what he called "idols" or tendencies to error.

from Of Studies
Francis Bacon

Some books are to be tasted, others to be swallowed, and some few to be chewed and digested; that is, some
books are to be read only in parts; others to be read, but not curiously; and some few to be read wholly, and with
diligence and attention. Some books also may be read by deputy, and extracts made of them by others; but that
would be only in the less important arguments, and the meaner sort of books, else distilled books are like
common distilled waters, flashy things. Reading maketh a full man; conference a ready man; and writing an exact
man.

2. John Bunyan (1628-1688) was an English Christian writer and preacher notable for his Christian allegory The
Pilgrim's Progress.

Allegory is story illustrating an idea or a moral principle in which objects and characters take on symbolic
meanings external to the narrative. Pilgrim’s Progress shows Christian tormented by spiritual anguish. A
spiritual guide named Evangelist visits Christian and urges him to leave the City of Destruction. Evangelist
claims that salvation can only be found in the Celestial City, known as Mount Zion. Christian embarks on a
journey and meets a number of other characters before he reaches the Celestial City.
3. John Milton (1608-74) was a Puritan poet who served Cromwell as Latin secretary; he is best known for his epic
poem Paradise Lost and its sequel Paradise Regained.
 Paradise Lost is an epic poem in blank verse that tells of the fall of the angels and of the creation of Adam and
Eve and their temptation by Satan in the Garden of Eden ("Of Man's first disobedience, and the fruit/ Of that
Holy Sonnets
forbidden tree . . . X
").
4. John Donne John Donne
(1572-1631) was the greatest of the metaphysical poets best remembered for his use of metaphysical
conceits in the Holy Sonnets.
Death, be not proud, though some have
 Metaphysical
called thee Poetry makes use of conceits or farfetched similes and metaphors intended to startle the
reader
Mighty and into an
dreadful, forawareness of the
thou art not so; relationships among things ordinarily not associated.
To the Virgins to Make Much of
2.
ForGeorge Herbert
those, whom thou(1593-1633),
think'st thou like
dostDonne, was both a metaphysical poet and an Anglican priest.Time Some of Herbert's
most effective poetry
overthrow, deals with man's thirst for God and with God's abounding love. Robert Herrick
Die not, poor Death, nor yet canst thou kill from Easter Wings
me. Gather ye rosebuds while ye may,
George Herbert
From rest and sleep, which but thy Old time is still a-flying:
picture[s] be, And this same flower that smiles to-
My tender age in sorrow did
Much pleasure, then from thee much more day
beginne:
must flow, To-morrow will be dying.
And still with sicknesses and
And soonest our best men with thee do The glorious lamp of heaven, the
shame
go, sun,
Thou didst so punish sinne,
Rest of their bones, and soul's delivery. The higher he's a-getting,
That I became Most thinne.
Thou'rt slave to Fate, chance, kings, and The sooner will his race be run,
desperate men, And nearer he's to setting.
With thee
And dost with poison, war, and sickness That age is best which is the first,
Let me combine,
dwell, When youth and blood are warmer;
And feel this day thy victorie,
And poppy, or charms can make us sleep But being spent, the worse, and
For, if I imp my wing on
as well, worst
thine,
And better than thy stroke; why swell'st Times still succeed the former.
Affliction shall advance the
thou then? Then be not coy, but use your time,
flight in me.
One short sleep past, we wake eternally, 10 And while ye may go marry:
And Death shall be no more; Death, thou For having lost but once your prime
shalt die. You may for ever tarry.
5. Andrew Marvell (1621-1678), Richard Crashaw (1612-1649), and Henry Vaughan (1622-1695) were other
metaphysical poets of merit. Marvell is famous for his well-loved lyric To His Coy Mistress.
6. Cavalier Poets include Thomas Carew (1595-1639), Richard Lovelace (1618-58), Sir John Suckling (1609-
42), and Robert Herrick (1591-1674). Cavalier poets preferred more straightforward expression. They value
elegance, and were part of a refined, courtly culture, but their poetry is often frankly erotic. Their strength was the
short lyric poem, and a favorite theme was carpe diem, "seize the day."

THE 18th CENTURY (RESTORATION)


A. Representative Writers
1. Jonathan Swift (1667-1745) is one of the great prose writers of all time.
 A Modest Proposal is a bitter pamphlet that ironically suggests that the Irish babies be specially fattened for
profitable sale as meat, since the English were eating the Irish people anyhow – by heavy taxation.
 Gulliver's Travels is a satire on human folly and stupidity. Swift said that he wrote it to vex the world rather
than to divert it. Most people, however, are so delightfully entertained by the tiny Lilliputians and by the huge
Brobdingnagians that they do not bother much with Swift's bitter satire on human pettiness or crudity.
2. Alexander Pope (1688-1744) published an exposition of the rules of the classical school in the form of a poem
An Essay on Criticism.
 The Rape of the Lock mockingly describes a furious fight between two families when a young man snips off a
lock of the beautiful Belinda's hair. Pope wrote in heroic couplets, a technique in which he has been
unsurpassed. In thought and form he carried 18th-century reason and order to its highest peak.
3. Thomas Gray (1716-71) wrote Elegy Written in a Country Churchyard, which is a collection of 18th-century
commonplaces expressing concern for lowly folk.
4. Henry Fielding (1707-54) is known for his Tom Jones, which tells the story of a young foundling who is driven
from his adopted home, wanders to London, and eventually, for all his suffering, wins his lady.
5. Laurence Sterne (1713-68) wrote Tristram Shandy, a collection of episodes with little organization but a wealth
of 18th-century humor.
6. Oliver Goldsmith (1728-74) wrote one of the best plays She Stoops to Conquer, one of the best poems The
Deserted Village, and one of the best novels The Vicar of Wakefield of the latter half of the 18th century.

THE ROMANTIC MOVEMENT IN ENGLAND


A. Romanticism
William Wordsworth and Samuel Taylor Coleridge presented and illustrated a liberating aesthetic. In the
Preface to Lyrical Ballads, they declared that “poetry should express, in genuine language, experience
as filtered through personal emotion and imagination; the truest experience was to be found in
”.nature
1. The most important tenets of Romanticism were belief in the importance of the individual, imagination, and intuition.
2. The Romanticists believed that all humans deserve the treatment to which human beings are by nature entitled. Every
human has a right to life, liberty, and equal opportunity.
3. The main tenets of Romanticism included a shift from faith in reason to faith in the senses, feelings, and imagination;
from interest in urban society and its sophistication to an interest in the rural and natural; from public, impersonal
poetry to subjective poetry; and from concern with the scientific and mundane to interest in the mysterious and
infinite.
4. Because of this concern for nature and the simple folk, authors began to take an interest in old legends, folk ballads,
antiquities, ruins, "noble savages," and rustic characters.
 Many writers started to give more play to their senses and to their imagination.
 They loved to describe rural scenes, graveyards, majestic mountains, and roaring waterfalls.
 They also liked to write poems and stories of such eerie or supernatural things as ghosts, haunted castles, fairies,
and mad folk.
B. Pre-Romantic Writers
1. Robert Burns (1759-96) is also known as the national poet of Scotland because he wrote not only in Standard
English, but also in the light Scot’s dialect.
 His nature lyrics are tenderly beautiful ('To a Mountain Daisy); his sentimental songs
are sung wherever young or old folks gather (Auld Lang Syne, Flow Gently Sweet Afton). His rich humor can
still be felt in Tam o' Shanter, To a Louse, and The Cotter's Saturday Night.
2. Gothic writers crafted stories of terror and imagination. Representative novels are The Castle of Otranto by Horace
Walpole; The Mysteries of Udolpho by Ann Radcliffe; and The Monk, by Matthew Gregory Lewis. All these novels
are filled with the machinery of sensationalism unreal characters, supernatural events, and overripe imagination.
The Sick Rose
3. Mary Wollstonecraft Shelley (1797-1851) followed Gothic tradition in her Frankenstein. William Blake
C. The Romanticists
1. William Blake (1757-1827) was both poet and artist. He not only wrote books, but he also O illustrated
ROSE, thouand art printed
sick!
them. He devoted his life to freedom and universal love. He was interested in children and animals the most innocent
The invisible worm,
of God's creatures. As he wrote in Songs of Innocence: When the voices of children are heard Thaton theingreen,
flies the night,
And laughing is heard on the hill, In the howling storm,
My heart is at rest within my breast,
And everything else is still. Has found out thy bed
11 Of crimson joy;
And his dark secret love
Does thy life destroy.
 He also wrote The Marriage of Heaven and Hell which
attacks hypocrisy, and Song of Experience, which presents a somber world, one
that is sick and diseased by lust and greed, with nature replaced by Churches,
factories and ale-houses.
2. Samuel Taylor Coleridge (1772-1834) put more wonder and mystery into beautiful
melodic verse than did. The strange, haunting supernaturalism of The Rime of the
Ancient Mariner and Christabel' have universal and irresistible appeal.
3. Charles Lamb (1775-1834) wrote the playful essay Dissertation on Roast Pig. He also
rewrote many of Shakespeare's plays into stories for children in Tales from
Shakespeare.
4. Sir Walter Scott (1771-1832) wrote poems and novels. The Lay of the Last
Minstrel and The Lady of the Lake are representative of Scott's poems. Between
1814 and 1832 Scott wrote 32 novels which include Guy Mannering and Ivanhoe
5. William Wordsworth (1770-1850), together with She Dwelt Among the Untrodden Ways
Coleridge, brought out a volume of verse, Lyrical Ballads, William Wordsworth
which signaled the beginning of English Romanticism. She dwelt among the untrodden ways
Wordsworth found beauty in the realities of nature, which Beside the springs of Dove,
he vividly reflects in the poems: The World is Too Much A Maid whom there were none to praise
with Us, I Wandered Lonely as a Cloud, She Dwelt And very few to love:
Among the Untrodden Ways, and She was a Phantom
of Delight. A violet by a mossy stone
6. Jane Austen (1775-1817) was a gifted writer of realistic Half hidden from the eye!
novels, but who experienced difficulty finding a publisher --Fair as a star, when only one
for her skillfully drawn portraits of English middle-class Is shining in the sky.
people. Pride and Prejudice is her best-known work. Her
other novels include: Northanger Abbey, Persuasion, She lived unknown, and few could know
Mansfield Park, Emma, and Sense and Sensibility. When Lucy ceased to be;
But she is in her grave, and, oh,
The difference to me!
D. The Younger Romanticists
1. George Gordon Byron (1788-1824) was an outspoken critic of the evils of his time. He hoped for human perfection,
but his recognition of man's faults led him frequently to despair and disillusionment. He is much remembered for his
poems: Childe Harold’s Pilgrimage, She Walks in Beauty, and The Prisoner of Chillon.
2. Percy Bysshe Shelley (1792-1822), together with John Keats, established the romantic verse as a poetic tradition.
 Many of his works are profound and meditative like Prometheus Unbound; others are exquisitely lyrical and
beautiful like The Cloud, To a Skylark, and Ode to the West Wind. Adonais, his tribute to Keats, ranks
among the greatest elegies.
 In Ode to the West Wind, Shelley shows an evocation of nature wilder and more spectacular than Wordsworth
described it.
3. John Keats (1795-1821) believed that true happiness From A Thing of Beauty is a Joy Forever
was to be found in art and natural beauty. John Keats
 His Ode to a Nightingale spoke of what Keats called
“negative capability,” describing it as the moment of A thing of beauty is a joy for ever:
artistic inspiration when the poet achieved a kind of self- Its loveliness increases; it will never
annihilation – arrived at that trembling, delicate Pass into nothingness; but still will keep
perception of beauty. A bower quiet for us, and a sleep
Full of sweet dreams, and health, and quiet
breathing.
ENGLISH LITERATURE OF THE VICTORIAN AGE
A. Major Victorian Poets - shifted from the extremely personal expression (or subjectivism) of the Romantic writers to
an objective surveying of the problems of human life.
1. Alfred Tennyson (1809-92) reflects his age especially in his idealism and his devotion to rather formal virtue. He
wrote seriously with a high moral purpose.
 Idylls of the King is a disguised study of ethical and social conditions. Locksley Hall, In Memoriam, and
Maud deal with conflicting scientific and social ideas.
2. Elizabeth Barrett Browning (1806-1861) wrote the most exquisite love poems of her time in Sonnets from the
Portuguese. These lyrics were written secretly while Robert Browning was courting her.
3. Robert Browning (1812-89) is best remembered for his dramatic monologues. My Last Duchess, Fra Lippo
Lippi, and Andrea del Sarto are excellent examples.
 Dramatic monologue is a long speech by an imaginary character used to expose pretense and reveal a
character’s inner self.
4. The Pre-Raphaelite Brotherhood is a group of painters and poets who rebelled against the sentimental and the
commonplace. They wished to revive the artistic standards of the time before the Italian painter Raphael.
 Dante Gabriel Rossetti (1828-1882). His Blessed Damozel and Sister Helen are typical of his highly
sensuous verse.
 Christina Georgina Rossetti (1830-1894), Gabriel’s sister, wrote one of the most fanciful poems in the
language, Goblin Market.
B. Victorian Novelists
1. Charles Dickens (1812-1870) became a master of local color in The Pickwick Papers. He is considered as
England's best-loved novelist. His works include: Great Expectations, Hard Times, Oliver Twist, A Christmas
Carol, A Tale of Two Cities.
2. William Makepeace Thackeray (1811-1863) disliked sham, hypocrisy, stupidity, false optimism, and self-seeking.
The result was satire on manners like Vanity Fair with its heroine, Becky Sharp.
3. Charlotte Bronte (1816-1855), Emily Bronte (1818-1848) and Anne Bronte (1820-1849) wrote novels romantic
novels.
 Charlotte's Jane Eyre and Emily's Wuthering Heights, especially, are powerful and intensely personal stories of
the private lives of characters isolated from the rest of the world.
4. George Eliot (1819-80) was one of England's greatest women novelists. She is famous for Silas Marner and
Middlemarch.
12
5. Thomas Hardy (1840-1928) is a naturalist writer who brought to fiction a philosophical attitude that resulted from
the new science.
 Hardy’s Wessex novels from The Return of the Native, Tess of d’Urbervilles, Mayor of Casterbridge to
Jude the Obscure sought to show the futility and senselessness of human’s struggle against the forces of
natural environment, social convention, and biological heritage.
6. Samuel Butler (1835-1902) believed that evolution is the result of the creative will rather than of chance selection.
His novel The Way of All Flesh explores the relationships between parents and children where he reveals that the
family restrains the free development of the child.
C. Romance and Adventure
1. Robert Louis Stevenson (1850-94) wrote stories in a light mood. His novels of adventure are exciting and
delightful: Treasure Island, Kidnapped, and The Master of Ballantrae.
 Stevenson also wrote David Balfour and The Strange Case of Dr. Jekyll and Mr. Hyde which endear him to
adult readers as well.
2. Rudyard Kipling (1865-1936) satirized the English military and administrative classes in India. He stirred the
emotions of the empire lovers through his delightful children's tales. He is known for Barrack Room Ballads,
Soldiers Three, The Jungle Books, and Captains Courageous.
3. Lewis Carroll (Charles Lutwidge Dodgson) (1832-98) combines fantasy and satire in Alice's Adventures in
Wonderland and Through a Looking Glass.
D. 19th-Century Drama
1. Oscar Wilde (1854-1900) is a poet and novelist who became famous for his Importance of Being Earnest.
2. George Bernard Shaw (1856-1950) wrote plays known for their attacks on Victorian prejudices and attitudes. Shaw
began to write drama as a protest against existing conditions slums, sex hypocrisy, censorship, and war. Because his
plays were not well received, Shaw wrote their now-famous prefaces.

MODERN ENGLISH LITERATURE


1. Early 20th-Century Prose
a. John Galsworthy (1867-1933) depicted the social life of an upper-class English family in The Forsyte Saga, a
series of novels which records the changing values of such a family.).
b. H.G. Wells (1866-1946) wrote science fiction like The Time Machine, The Island of Dr. Moreau, and The
War of the Worlds. He also wrote social and political satires criticizing the middle-class life of England. A good
example is Tono-Bungay which attacks commercial advertising.
c. Joseph Conrad (1857-1924) wrote remarkable novels as The Nigger of the Narcissus and Lord Jim where he
depicts characters beset by obsessions of cowardice, egoism, or vanity.
d. E.M. Forster (1879-1970) is a master of traditional plot. His characters are ordinary persons out of middle-class
life. They are moved by accident because they do not know how to choose a course of action. He is famous for A
Passage to India, a novel that shows the lives of Englishmen in India.
2. Early 20th-Century Poetry
a. A.E. Housman (1859-1936) was an anti-Victorian who echoed the pessimism found in Thomas Hardy. In his
Shropshire Lad, nature is unkind; people struggle without hope or purpose; boys and girls laugh, love, and are
untrue.
When I Was One and Twenty When I was one-and-twenty
A.E. Housman I heard him say again,
‘The heart out of the bosom
WHEN I was one-and-twenty Was never given in vain;
I heard a wise man say,
‘Give crowns and pounds and guineas ’Tis paid with sighs a plenty
But not your heart away; And sold for endless rue.’
And I am two-and-twenty,
Give pearls away and rubies And oh, ’tis true, ’tis true.
But keep your fancy free.’
But I was one-and-twenty,
No use to talk to me.

b.
William Butler Yeats (1865-1939), John Millington Synge (1871-1909), and Lord Dunsany (1878-1957)
worked vigorously for the Irish cause. All were dramatists and all helped found the famous Abbey Theatre.
3. Writers after the World Wars
World War I brought discontent and disillusionment. Men were plunged into gloom at the knowledge that "progress"
had not saved the world from war. In fiction there was a shift from novels of the human comedy to novels of
characters. Fiction ceased to be concerned with a plot or a forward-moving narrative. Instead it followed the
twisted, contorted development of a single character or a group of related characters
a. William Somerset Maugham (1874-1965) focused on the alienation and despair of drifters. His Of Human
Bondage portrays Philip Carey struggling against self-consciousness and embarrassment because of his cub-
foot.
b. D.H. Lawrence (1885-1930) explored highly psychological themes as human desire, sexuality, and instinct
alongside the dehumanizing effects of modernity and industrialization in such great novels as Sons and Lovers,
Women in Love, The Plumed Serpent, and Lady Chatterley’s Lover.
c. James Joyce (1882-1941) was an Irish expatriate noted for his experimental use of the interior monologue
and the stream of consciousness technique in landmark novels as Ulysses, Finnegans Wake, and in his
semi-autobiographical novel The Portrait of the Artist as a Young Man’.
 Stream of consciousness is a technique pioneered by Dorothy Richardson, Virginia Woolf and James Joyce.
It presents the thoughts and feelings of a character as they occur.
 Joyce’s A Portrait of the Artist as a Young Man is one of the most notable bildungs-roman in English
literature. A bildungsroman is a novel of formation or development in which the protagonist transforms
from ignorance to knowledge, innocence to maturity.

13
d. Virginia Woolf (1882-1941) also believed that reality, or consciousness, is a stream. Life, for both reader and
characters, is immersion in the flow of that stream. Mrs. Dalloway and To the Lighthouse are among her best
works.
e. Aldous Huxley (1894-1963) wrote Point Counter Point, Brave New World, and After Many a Summer
Dies the Swan where he showed his cynicism of the contemporary world.
f. William Golding (born 1911) was awarded the Nobel Prize for literature in 1983. His first novel, Lord of
the Flies tells of a group of schoolboys who revert to savagery when isolated on an island. In the novel, Golding
explores naturalist and religious themes of original sin.
g. George Orwell (1903-50) is world-renown, for the powerful anti-Communist satire Animal Farm. This was
followed in 1949 with an anti-totalitarian novel entitled Nineteen Eighty-Four.
h. Graham Greene (1904-91) is known for novels of highly Catholic themes like Brighton Rock, The Heart of
the Matter, The End of the Affair and The Power and the Glory. Among his better-known later novels are
The Quiet American, Our Man in Havana, A Burnt-Out Case, The Human Factor, and Monsignor
Quixote.
i. Kingsley Amis is considered by many to be the best of the writers to emerge from the 1950s. The social
discontent he expressed made Lucky Jim famous in England. Lucky Jim is the story of Jim Dixon, who rises
from a lower-class background only to find all the positions at the top of the social ladder filled.
j. Anthony Burgess (born 1917) was a novelist whose fictional exploration of modern dilemmas combines wit,
moral earnestness, and touches of the bizarre. He is known for A Clockwork Orange. His other novels include
Enderby Outside, Earthly Powers, The End of the World News, and The Kingdom of the Wicked.
k. Doris Lessing (born 1919) is a Zimbabwean-British writer, famous for novels The Grass is Singing and The
Golden Notebook. She won the Nobel Prize for Literature in 2007.
l. Salman Rushdie is a British-Indian novelist and essayist noted for his Midnight's Children and The Satanic
Verses which prompted Iran's Ayatollah Khomeini to issue a fatwa against him, because Muslims considered the
book blasphemous. In July 2008 Midnight's Children won a public vote to be named the Best of the Booker, the
best novel to win the Booker Prize in the award's 40-year history.

AMERICAN LITERATURE
A. Early American and Colonial Period to 1776
 American literature begins with the orally transmitted myths, legends, tales, and lyrics (always songs) of Indian
cultures. There was no written literature among the more than 500 different Indian languages and tribal cultures that
existed in North America before the first Europeans arrived.
 Indian stories are characterized by the following:
o reverence for nature as a spiritual as well as physical mother
o nature is rendered alive and endowed with spiritual forces
o main characters may be animals or plants, often totems associated with a tribe, group, or individual
o Accounts of migrations and ancestors abound, as do vision or healing songs and tricksters' tales.
 The songs or poetry, like the narratives, range from the sacred to the light and humorous: There are lullabies, war
chants, love songs, and special songs for children's games, gambling, various chores, magic, or dance ceremonials.
 Examples of almost every oral genre can be found in American Indian literature: lyrics, chants, myths, fairy tales,
humorous anecdotes, incantations, riddles, proverbs, epics, and legendary histories. Certain creation stories are
particularly popular.
B. THE LITERATURE OF EXPLORATION
1. Christopher Columbus the famous Italian explorer, funded by the Spanish rulers Ferdinand and Isabella, wrote the
"Epistola," printed in 1493 which recounts his voyages.
2. Captain John Smith led the Jamestown colony and wrote the famous story of the Indian maiden, Pocahontas.
C. COLONIAL PERIOD IN NEW ENGLAND
1. William Bradford (1590-1657) wrote Of Plymouth Plantation and the first document of colonial self-
governance in the English New World, the Mayflower Compact.
2. Anne Bradstreet (c. 1612-1672) wrote the first published
book of poems by an American which was also the first American To my Dear and Loving Husband
book to be published by a woman. Anne Bradstreet
 She wrote long, religious poems on conventional subjects,
but she is well loved for her witty poems on subjects from If ever two were one, then surely we.
daily life and her warm and loving poems to her husband If ever man were lov'd by wife, then thee.
and children. If ever wife was happy in a man,
 She was inspired by English metaphysical poetry, and her Compare with me, ye women, if you can.
book The Tenth Muse Lately Sprung Up in America I prize thy love more than whole Mines of gold
(1650) shows the influence of Edmund Spenser, Philip Or all the riches that the East doth hold.
Sidney, and other English poets as well. My love is such that Rivers cannot quench,
3. Edward Taylor (c. 1644-1729) was an intense, brilliant poet, Nor ought but love from thee give recompense.
teacher and minister who sailed to New England in 1668 rather Thy love is such I can no way repay.
than take an oath of loyalty to the Church of England. The heavens reward thee manifold, I pray.
 He wrote a variety of verse: funeral elegies, lyrics, a Then while we live, in love let's so persever
medieval "debate," and a 500-page Metrical History of That when we live no more, we may live ever.
Christianity (mainly a history of martyrs). His best works,
according to modern critics, are the series of short
Preparatory Meditations.
4. Jonathan Edwards (1703-1758) was molded by his extreme sense of duty and by the rigid Puritan environment,
which conspired to make him defend strict and gloomy Calvinism from the forces of liberalism springing up around
him. He is best known for his frightening, powerful sermon, Sinners in the Hands of an Angry God.
 Puritans refer to two distinct groups: "separating" Puritans, such as the Plymouth colonists, who believed that
the Church of England was corrupt and that true Christians must separate themselves from it; and non-
separating Puritans, such as those in Massachusetts Bay Colony, who believed in reform but not separation.
 Puritans believed in God’s ultimate sovereignty in granting grace and salvation; therefore, their lives center on
three important covenants – covenants of Works, Grace, and Redemption.
D. THE AMERICAN ENLIGHTENMENT

14
Enlightenment thinkers and writers were devoted to the ideals of justice, liberty, and equality as the natural rights of
man. Thus, the18th-century American Enlightenment was a movement marked by -
 an emphasis on rationality rather than tradition,
 scientific inquiry instead of unquestioning religious dogma, and
 Representative government in place of monarchy.
1. Benjamin Franklin (1706-1790) was America's "first great man of letters," who embodied the Enlightenment ideal
of humane rationality.
 Franklin was a leading author and printer, satirist, political theorist, politician, scientist, inventor, civic activist,
statesman, and diplomat.
 He was an important figure at the 1787 convention at which the U.S. Constitution was drafted. In his later years,
he was president of an antislavery association and one of his last efforts was to promote universal public
education.
 He used the pseudonym Poor Richard or Richard Saunders in Poor Richard’s Almanack – a yearly almanac he
released from 1732-1758. The almanac was a repository of Franklin’s proverbs and aphorisms.
2. Thomas Paine (1737-1809) is known for his political pamphlets. His pamphlet Common Sense sold over 100,000
copies in the first three months of its publication. He wrote the famous line, "The cause of America is in a great
measure the cause of all mankind."
3. Philip Freneau (1752-1832) was the poet of the American Revolution who incorporated the new stirrings of
European Romanticism in his lyric The Wild Honeysuckle.
4. Washington Irving (1789-1859) became a cultural and diplomatic ambassador to Europe, like Benjamin Franklin
and Nathaniel Hawthorne.
 With the help of friends, he was able to publish his Sketch Book (1819-1820) simultaneously in England and
America, obtaining copyrights and payment in both countries. The Sketch Book of Geoffrye Crayon (Irving's
pseudonym) contains his two best-remembered stories, Rip Van Winkle and The Legend of Sleepy Hollow.
5. James Fenimore Cooper (1789-1851) wrote the Leather Stocking tales in which he introduced his renowned
character Natty Bumppo, who embodies his vision of the frontiersman as a gentleman, a Jeffersonian "natural
aristocrat."
 Natty Bumppo is the first famous frontiersman in American literature and the literary forerunner of countless
cowboy and backwoods heroes.
6. Phillis Wheatley (c. 1753-1784) is the first African-American author who wrote of religious themes. Just like that of
Philip Freneau, her style is neoclassical.
 Among her best-known poems are To S.M., a Young African Painter, on Seeing His Works and On Being
Brought from Africa to America. These poems boldly confront white racism and assert spiritual equality.
E. THE ROMANTIC PERIOD, 1820-1860
 The Romantic Movement, which originated in Germany but quickly spread to England, France, and beyond,
reached America around the year 1820, some 20 years after William Wordsworth and Samuel Taylor Coleridge had
revolutionized English poetry by publishing Lyrical Ballads.
 Romanticism in America coincided with the period of national expansion and the discovery of a distinctive
American voice.
 Romantic ideas centered on art as inspiration, the spiritual and aesthetic dimension of nature, and metaphors of
organic growth.
 Art, rather than science, could best express universal truth. The Romantics underscored the importance of expressive
art for the individual and society. In his essay The Poet, Ralph Waldo Emerson asserts:
For all men live by truth, and stand in need of expression. In love, in art, in avarice, in politics, in
labor, in games, we study to utter our painful secret. The man is only half himself, the other half
is his expression.
 The development of the self became a major theme; self- awareness a primary method. The idea of "self" - which
suggested selfishness to earlier generations - was redefined. New compound words with positive meanings emerged:
"self-realization," "self-expression," "self- reliance."
 As the unique, subjective self became important, so did the realm of psychology. Exceptional artistic effects and
techniques were developed to evoke heightened psychological states. The "sublime" -- an effect of beauty in
grandeur, produced feelings of awe, reverence, vastness, and a power beyond human comprehension.
 Romanticism was affirmative and appropriate for most American poets and creative essayists. America's vast
mountains, deserts, and tropics embodied the sublime. The Romantic spirit seemed particularly suited to American
democracy.
Transcendentalists
 The Transcendentalist movement was a reaction against 18th century rationalism and a manifestation of the
general humanitarian trend of 19th century thought.
 The movement was based on the belief in the unity of the world and God.
 The doctrine of self- reliance and individualism developed through the belief in the identification of the individual
soul with God.
1. Ralph Waldo Emerson (1803-1882) was a leading exponent of the transcendentalist movement who called for the
birth of American individualism inspired by nature.
 In his essay Self-Reliance, Emerson remarks: "A foolish consistency is the hobgoblin of little minds."
 Most of his major ideas – the need for a new national vision, the use of personal experience, the notion of the
cosmic Over-Soul, and the doctrine of compensation – are suggested in his first publication, Nature.
2. Henry David Thoreau (1817-1862) wrote Walden, or Life in the Woods, which was the result of two years, two
months, and two days (from 1845 to 1847) he spent living in a cabin he built at Walden Pond on property owned by
Emerson.
 In Walden, Thoreau not only tests the theories of transcendentalism, but he also re-enacts the collective
American experience of the 19th century by living on the frontier.
 He also wrote Civil Disobedience, with its theory of passive resistance based on the moral necessity for the just
individual to disobey unjust laws. This was an inspiration for Mahatma Gandhi's Indian independence movement
and Martin Luther King's struggle for black Americans' civil rights in the 20th century.
3. Walt Whitman (1819-1892) incorporated both
transcendentalist and realist ideas in his works. He From Song of Myself
championed the individual and the country's democratic spirit Walt Whitman
in his Leaves of Grass.
I CELEBRATE myself, and sing myself,
And what I assume you shall assume,
15 For every atom belonging to me as good belongs to
you.
 Leaves of Grass, which he rewrote and revised
throughout his life, contains Song of Myself, the
strongest evocation of the transcend list ideals.

I Heard a Fly Buzz When I Died


Emily Dickinson
I heard a fly buzz when I died; The stillness round my form 4. Emily Dickinson (1830-1886) was a radical
was like the stillness in the air between the heaves of storm. individualist who found deep inspiration in the birds,
The eyes beside had wrung them dry, and breaths were animals, plants, and changing seasons of the New
gathering sure England countryside. She wrote 1,775 poems but only
For that last onset, when the king be witnessed in his power. I one was published in her lifetime.
willed my keepsakes, signed away  She shows a terrifying existential awareness. Like
What portion of me I could make assignable, and then Poe, she explores the dark and hidden part of the
there interposed a fly, mind, dramatizing death and the grave.
With blue, uncertain, stumbling buzz, between the light and
me; And then the windows failed, and then
I could not see to see.

The Brahmin Poets


Boston Brahmin poets refer to the patrician, Harvard-educated literati who sought to fuse American and European
traditions in their writings.
1. Henry Wadsworth Longfellow (1807-1882) was responsible for the misty, ahistorical, legendary sense of the
past that merged American and European traditions.
 He wrote three long narrative poems popularizing native legends in European meters Evangeline, The Song of
Hiawatha, and The Courtship of Miles Standish.
 He also wrote short lyrics like The Jewish Cemetery at Newport, My Lost Youth, and The Tide Rises, The
Tide Falls.
2. Oliver Wendell Holmes (1809-1894) was a physician and professor of anatomy and physiology at Harvard. Of the
Brahmin poets, he is the most versatile. His works include collections of humorous essays (The Autocrat of the
Breakfast-Table), novels (Elsie Venner), biographies (Ralph Waldo Emerson), and verses (The Deacon's
Masterpiece, or The Wonderful One-Hoss Shay).
The Romantic Period, 1820-1860: Fiction
1. Nathaniel Hawthorne (1804-1864) set his stories in Puritan New England. His greatest novels, The Scarlet
Letter and The House of the Seven Gables; and his best-known shorter stories The Minister's Black Veil,
Young Goodman Brown, and My Kinsman, Major Molineux, all highlight the Calvinistic obsession with morality,
sexual repression, guilt and confession, and spiritual salvation.
2. Herman Melville (1819-1891) went to sea when he was just 19 years old. His interest in sailors' lives grew
naturally out of his own experiences, and most of his early novels grew out of his voyages.
 Moby-Dick is Melville's masterpiece. It is the epic story of the whaling ship Pequod and its "ungodly, god-like
man," Captain Ahab, whose obsessive quest for the white whale Moby-Dick leads the ship and its men to
destruction.
3. Edgar Allan Poe (1809-1849) refined the short story genre and invented detective fiction. Many of his stories
prefigure the genres of science fiction, horror, and fantasy so popular today.
 His famous works The Cask of Amontillado, Masque of the Red Death, The Fall of the House of Usher,
Purloined Letter, and the Pit and the Pendulum, all center on the mysterious and the macabre.
 He also wrote poetry like Anabel Lee, The Raven, and The Bell.
4. Sojourner Truth (c.1797-1883) epitomized the endurance of the women reformers.
 Born a slave in New York, she escaped from slavery in 1827, settling with a son and daughter in the supportive
Dutch-American Van Wagener family, for whom she worked as a servant.
 She worked with a preacher to convert prostitutes to Christianity and lived in a progressive communal home. She
was christened "Sojourner Truth" for the mystical voices and visions she began to experience. To spread the
truth of these visionary teachings, she sojourned alone, lecturing, singing gospel songs, and preaching
abolitionism through many states over three decades
5. Harriet Beecher Stowe (1811-1896) wrote Uncle Tom's Cabin; or, Life Among the Lowly which became the most
popular American book of the 19 th Century. Its passionate appeal for an end to slavery in the United States inflamed
the debate that, within a decade, led to the U.S. Civil War (1861-1865).
 Uncle Tom, the slave and central character, is a true Christian martyr who labors to convert his kind master, St.
Clare, prays for St. Clare's soul as he dies, and is killed defending slave women.
 Slavery is depicted as evil not for political or philosophical reasons but mainly because it divides families,
destroys normal parental love, and is inherently un-Christian.

F. THE RISE OF REALISM: 1860-1914


1. Samuel Clemens (Mark Twain) (1835-1910)
 Samuel Clemens, better known by his pen name of Mark Twain, grew up in the Mississippi River frontier town of
Hannibal, Missouri.
 Ernest Hemingway's famous statement that all of American literature comes from one great book, Twain's
Adventures of Huckleberry Finn, indicates this author's towering place in the tradition.
 Twain's style is vigorous, realistic, colloquial American speech, gave American writers a new appreciation of their
national voice.
 Huckleberry Finn has inspired countless literary interpretations. Clearly, the novel is a story of death, rebirth,
and initiation. The escaped slave, Jim, becomes a father figure for Huck; in deciding to save Jim, Huck grows
morally beyond the bounds of his slave-owning society. It is Jim's adventures that initiate Huck into the
complexities of human nature and give him moral courage.
2. Bret Harte (1836-1902) is remembered as a local colorist and author of adventurous stories such as The Luck of
Roaring Camp and The Outcasts of Poker Flat set along the western mining frontier.
3. Henry James (1843-1916) wrote that art, especially literary art, "makes life, makes interest, makes importance."
 With Twain, James is generally ranked as the greatest American novelist of the second half of the 19th century.

16
 James is noted for his "international theme" -- that is, the complex relationships between naive Americans and
cosmopolitan Europeans, which he explored in the novels The American, Daisy Miller, and a masterpiece, The
Portrait of a Lady.
4. Edith Wharton (1862-1937) descended from a wealthy family in New York society and saw firsthand the decline of
this cultivated group and, in her view, the rise of boorish, nouveau-riche business families. This social transformation
is the background of many of her novels.
 Wharton's best novels include The House of Mirth, The Custom of the Country, Summer, The Age of
Innocence, and the novella Ethan Frome.
5. Stephen Crane (1871-1900) was a journalist who also wrote fiction, essays, poetry, and plays.
 Crane saw life at its rawest, in slums and on battlefields. His short stories like The Open Boat, The Blue Hotel,
and The Bride Comes to Yellow Sky exemplify such realism.
 He wrote a haunting Civil War novel, The Red Badge of Courage which explores the psychological turmoil of a
self-confessed coward.
 Crane's Maggie: A Girl of the Streets is one of the best naturalistic American novels. It is the harrowing story
of a poor, sensitive young girl whose alcoholic parents utterly fail her. In love and eager to escape her violent
home life, she allows herself to be seduced into living with a young man, who soon deserts her. When her self-
righteous mother rejects her, Maggie becomes a prostitute to survive, but soon commits suicide out of despair.
6. Jack London (1876-1916) is a naturalist who set his collection of stories, The Son of the Wolf in the Klondike
region of Alaska and the Canadian Yukon. His best-sellers The Call of the Wild and The Sea-Wolf made him the
highest paid writer in the United States of his time.
7. Theodore Dreiser (1871-1945) explores the dangers of the American dream in his 1925 work An American
Tragedy, The novel relates, in great detail, the life of Clyde Griffiths, who grows up in great poverty in a family of
wandering evangelists, but dreams of wealth and the love of beautiful women.
 An American Tragedy is a reflection of the dissatisfaction, envy, and despair that afflicted many poor and
working people in America's competitive, success-driven society. As American industrial power soared, the
glittering lives of the wealthy in newspapers and photographs sharply contrasted with the drab lives of ordinary
farmers and city workers.
 Muckraking novels used eye-catching journalistic techniques to depict harsh working conditions and
oppression. Populist Frank Norris's The Octopus exposed big railroad companies, while socialist Upton Sinclair's
The Jungle painted the squalor of the Chicago meat-packing houses. Jack London's dystopia The Iron Heel
anticipates George Orwell's 1984 in predicting a class war and the takeover of the government.
8. Willa Cather (1873-1947) grew up on the Nebraska prairie among pioneering immigrants - later immortalized in O
Pioneers!, My Antonia, and her well-known story Neighbour Rosicky.
 During her lifetime she became increasingly alienated from the materialism of modern life and wrote of
alternative visions in the American Southwest and in the past.
 Death Comes for the Archbishop evokes the idealism of two 16th-century priests establishing the Catholic
Church in the New Mexican desert.
9. Carl Sandburg (1878-1967) was a poet, historian, biographer, novelist, musician, Fog
essayist, but a journalist by profession. Carl Sandburg
 To many, Sandburg was a latter-day Walt Whitman, writing expansive, evocative
urban and patriotic poems and simple, childlike rhymes and ballads. The fog comes
10. Edwin Arlington Robinson (1869-1935) is the best U.S. poet of the late 19th on little cat feet.
century. Unlike Masters, Robinson uses traditional metrics.
It sits looking
 Some of the best known of Robinson's dramatic monologues are Luke
over harbor and city
Havergal, about a forsaken lover; Miniver Cheevy, a portrait of a romantic
on silent haunches
dreamer; and Richard Cory, a somber portrait of a wealthy man who commits
and then moves on.
suicide.

G. MODERNISM AND EXPERIMENTATION: 1914-1945


1. Many historians have characterized the period between the two world wars as the United States' traumatic "coming
of age."
2. Gertrude Stein termed this age as the "Period of the Lost Generation." Many young Americans lost their sense of
identity because of the instability of traditional structure of values brought about by the wars and the growing
industrialization of cities.
3. The world depression of the 1930s affected most of the population of the United States. Workers lost their jobs, and
factories shut down; businesses and banks failed; farmers, unable to harvest, transport, or sell their crops, could not
pay their debts and lost their farms.
4. Freudian psychology and to a lesser extent Marxism (like the earlier Darwinian theory of evolution) became popular.
5. In literature, vision and viewpoint became an essential aspect of the modernist novel. No longer was it sufficient to
write a straightforward third-person because the way the story was told became as important as the story itself.
6. Henry James, William Faulkner, and many other American writers experimented with fictional points of view.
James often restricted the information in the novel to what a single character would have known. Faulkner's novel
The Sound and the Fury (1929) breaks up the narrative into four sections, each giving the viewpoint of a different
character (including a mentally retarded boy).
7. To analyze such modernist novels and poetry, New Criticism arose in the United States.

H. POETRY 1914-1945: EXPERIMENTS IN FORM


1. Ezra Pound (1885-1972) was one of the most influential American poets of this century. His poetry is best known
for its clear, visual images, fresh rhythms, and muscular, intelligent, unusual lines, such as, in Canto LXXXI, "The ant's a
centaur in his dragon world," or in poems inspired by Japanese haiku, such as "In a Station of the Metro" (1916):
The apparition of these faces in the crowd;
Petals on a wet, black bough.
3. T.S. Eliot (1888-1965) wrote influential essays and dramas, and championed the importance of literary and social
traditions for the modern poet. As a critic, Eliot is best remembered for his formulation of the "objective
correlative," as a means of expressing emotion through "a set of objects, a situation, a chain of events" that would
be the "formula" of that particular emotion.
For I have known them all already, known them all:
Have known the evenings, mornings, afternoons, The Love Song of J. Alfred Prufrock embodies
I have measured out my life with coffee spoons; this approach, when the ineffectual, elderly Prufrock
I know the voices dying with a dying fall 17 thinks to himself that he has "measured out his life in
Beneath the music from a farther room. coffee spoons," using coffee spoons to reflect a
So how should I presume? humdrum existence and a wasted lifetime.
3. Robert Frost (1874-1963) combines sound and sense in his frequent use of rhyme and images. Frost's poems are
often deceptively simple but suggest a deeper meaning.

Stopping By Woods on a Snowy


Evening Between the woods and frozen
lakeThe darkest evening of the year.
He gives his harness bells a shake
Whose woods these are I think I know.
To ask if there is some mistake.
His house is in the village though;
The only other sound's the sweep
He will not see me stopping here
Of easy wind and downy flake.
To watch his woods fill up with snow.
The woods are lovely, dark and deep.
My little horse must think it queer
But I have promises to keep,
To stop without a farmhouse near

4. Wallace Stevens (1879-1955) lived a double life, one as an insurance business The Red Wheelbarrow
executive, another as a renowned poet. His associates in the insurance company did William Carlos Williams
not know that he was a major poet.
 Some of his best known poems are "Sunday Morning," "Peter Quince at the so much depends
Clavier," "The Emperor of Ice-Cream," "Thirteen Ways of Looking at a upon
Blackbird," and "The Idea of Order at Key West."
 Stevens's poetry dwells upon themes of the imagination, the necessity for a red wheel
aesthetic form, and the belief that the order of art corresponds with an order in barrow
nature. His vocabulary is rich and various: He paints lush tropical scenes but
also manages dry, humorous, and ironic vignettes. glazed with rain
5. William Carlos Williams (1883-1963) championed the use of colloquial speech; water
his ear for the natural rhythms of American English helped free American poetry from
the iambic meter that had dominated English verse since the Renaissance. beside the white
 His sympathy for ordinary working people, children, and everyday events in chickens.
modern urban settings make his poetry attractive and accessible. The Red
Wheelbarrow, like a Dutch still life, finds interest and beauty in everyday
objects.
 He termed his work "objectivist" to suggest the importance of concrete, visual objects. His work influenced the
"Beat" writing of the early 1950s.
 Beat Generation refers to a group of American writers who became popular in the 1950s and who popularized
the “Beatniks" culture. The “Beatniks” rejected mainstream American values, experimented with drugs and
alternate forms of sexuality, and focused on Eastern spirituality.
 The major works of Beat writing are Allen Ginsberg's Howl, William S. Burroughs's Naked Lunch and Jack
Kerouac's On the Road.
6. Edward Estlin Cummings (1894-1962), commonly known as e.e. cummings, wrote innovative verse distinguished
for its humor, grace, celebration of love and eroticism, and experimentation with punctuation and visual format on the
page.
8. Langston Hughes (1902-1967) embraced African- American jazz rhythms in his works. He was one of the leaders
of the Harlem Renaissance responsible for the flowering of African-American culture and writings.

I. PROSE WRITING, 1914-1945: AMERICAN REALISM


1. F. Scott Fitzgerald (1896-1940) is known for novels whose protagonists are disillusioned by the great American
dream.
 The Great Gatsby focuses on the story of Jay Gatsby who discovers the devastating cost of success in terms of
personal fulfillment and love.
 Tender Is the Night talks of a young psychiatrist whose life is doomed by his marriage to an unstable woman.
 The Beautiful and the Damned explores the self-destructive extravagance of his times
2. Ernest Hemingway (1899-1961) received the Nobel Prize in 1954 for his The Old Man and the Sea – a short
poetic novel about a poor, old fisherman who heroically catches a huge fish devoured by sharks. This also won for
him the Pulitzer Prize in 1953
 Hemingway wrote of war, death, and the "lost generation" of cynical survivors. His characters are not dreamers
but tough bullfighters, soldiers, and athletes. If intellectual, they are deeply scarred and disillusioned.
3. William Faulkner (1897-1962) experimented with narrative chronology, different points of view and voices
(including those of outcasts, children, and illiterates), and a rich and demanding baroque style built of extremely long
sentences full of complicated subordinate parts.
 Created an imaginative landscape, Yoknapatawpha County, mentioned in numerous novels, along with several
families with interconnections extending back for generations.
 His best works include The Sound and the Fury and As I Lay Dying, two modernist works experimenting with
viewpoint and voice to probe southern families under the stress of losing a family member;
 Faulkner's themes are southern tradition, family, community, the land, history and the past, race, and the
passions of ambition and love.
4. Sinclair Lewis (1885-1951) is the first American to win the Nobel Prize for Literature in 1930.
 Lewis's Main Street satirized the monotonous, hypocritical small-town life in Gopher Prairie, Minnesota. His
incisive presentation of American life and his criticism of American materialism, narrowness, and hypocrisy
brought him national and international recognition.
 In 1926, he was offered and declined a Pulitzer Prize for Arrowsmith, a novel tracing a doctor's efforts to
maintain his medical ethics amid greed and corruption.

18
5. John Steinbeck (1902-1968) received the Nobel Prize for Literature in 1963 for his realist novel The Grapes of
Wrath, the story of a poor Oklahoma family that loses its farm during the Depression and travels to California to
seek work.
6. Sylvia Plath (1932-1963) was an American poet, novelist, short story and children’s author. She became famous
for her semi-autobiographical novel The Bell Jar, which pictures a woman trapped between the dictates of marriage,
mother, and wifehood and the demands of a creative spirit that.
 Confessional poetry was popularized by Robert Lowell, Richard Snodgrass, Anne Sexton, and Sylvia Plath. It is
a kind of poetry which reveals the poet’s personal life in poems about illnesses, sexuality, and despondence.
7. Richard Wright (1908-1960) was the first African-American novelist to reach a general audience, despite his little
education. He depicted his harsh childhood as a colored American in one of his best books, his autobiography, Black
Boy. He later said that his sense of deprivation, due to racism, was so great that only reading kept him alive.
8. Zora Neale Hurston (1903-1960) is known as one of the lights of the Harlem Renaissance. She first came to New
York City at the age of 16 - having arrived as part of a traveling theatrical troupe.
 Her most important work, Their Eyes Were Watching God, is a moving, fresh depiction of a beautiful mulatto
woman's maturation and renewed happiness as she moves through three marriages.
9. Eugene O'Neill (1888-1953) is the first American playwright to be honored with the Nobel Prize for Literature in
1936.
 O'Neill's earliest dramas concern the working class and poor, but his later works explore subjective realms, such
as obsessions, sex and other Freudian themes.
 His play Desire Under the Elms recreates the passions hidden within one family; The Great God Brown
uncovers the unconsciousness of a wealthy businessman; and his Strange Interlude, a winner of the Pulitzer
Prize, traces the tangled loves of one woman.
 O'Neill continued to explore the Freudian pressures of love and dominance within families in a trilogy of plays
collectively entitled Mourning Becomes Electra, based on the classical Oedipus trilogy by Sophocles.
10. Thornton Wilder (1897-1975) is known for his plays Our Town and The Skin of Our Teeth, and for his novel The
Bridge of San Luis Rey.
 Our Town has all the elements of sentimentality and nostalgia – the archetypal traditional small country town,
the kindly parents and mischievous children, the young lovers.
 It shows Wilder’s innovative elements such as ghosts, voices from the audience, and daring time shifts.
11. Arthur Miller (1915- ) is New York-born dramatist-novelist-essayist-biographer.
 He reached his personal pinnacle in 1949 with Death of a Salesman, a study of man's search for merit and
worth in his life and the realization that failure invariably looms.
 Miller also wrote All My Sons and The Crucible – both political satires.
12. Tennessee Williams (1911-1983) focused on disturbed emotions and unresolved sexuality within families - most of
them southern.
 As one of the first American writers to live openly as a homosexual, Williams explained that the sexuality of his
tormented characters expressed their loneliness. He was known for incantatory repetitions, a poetic southern
diction, weird Gothic settings, and Freudian exploration of sexual desire. He became famous for his The Glass
Menagerie and A Streetcar Named Desire.

K. THE AFFLUENT BUT ALIENATED 1950s


 The 1950s saw the delayed impact of modernization and technology in everyday life left over from the 1920s -
before the Great Depression.
 World War II brought the United States out of the Depression, and the 1950s provided most Americans with time
to enjoy long-awaited material prosperity.
 Loneliness at the top was a dominant theme. The 1950s actually was a decade of subtle and pervasive
stress. Novels by John O'Hara, John Cheever, and John Updike explore the stress lurking in the shadows of
seeming satisfaction.
 Some of the best works portray men who fail in the struggle to succeed, as in Arthur Miller's Death of a
Salesman and Saul Bellow's novella Seize the Day.
 Some writers went further by following those who dropped out, as did J.D. Salinger in The Catcher in the Rye,
Ralph Ellison in Invisible Man, and Jack Kerouac in On the Road.
 Philip Roth published a series of short stories reflecting his own alienation from his Jewish heritage – Goodbye,
Columbus.
 The fiction of American Jewish writers Bellow, Bernard Malamud, and Isaac Bashevis Singer – are most noted for
their humor, ethical concern, and portraits of Jewish communities in the Old and New Worlds.
1. Ralph Waldo Ellison (1914-1994) is known for his one highly-acclaimed book the Invisible Man (1952) which is a
story of a black man who lives a subterranean existence in a hole brightly illuminated by electricity stolen from a
utility company. The book recounts his grotesque, disenchanting experiences.
2. Katherine Anne Porter (1890-1980) created fiction organized around a single narrator telling the story from a
consistent point of view. Her first success, the story Flowering Judas, was set in Mexico during the revolution.
3. Eudora Welty (1909-2001) modeled after Katherine Ann Porter, but she is more interested in the comic and
grotesque characters like the stubborn daughter in her short story Why I Work at the P.O., who moves out of her
house to live in a tiny post office.
5. Saul Bellow (1915-2005) received the Nobel Prize for Literature in 1976.
 Bellow's Seize the Day is a brilliant novella noted for its excellence and brevity. It
centers on a failed businessman, Tommy Wilhelm, who tries to hide his feelings of inadequacy by presenting a
good front. Seize the Day sums up the fear of failure that plagues many Americans.
6. J.D. Salinger (1919- ) achieved huge literary success with the publication of his novel The Catcher in the Rye
(1951).
 The novel centers on a sensitive 16-year-old, Holden Caulfield, who flees his elite
boarding school for the outside world of adulthood, only to become disillusioned by its materialism and
phoniness. When asked what he would like to be, Caulfield answers "the catcher in the rye," In his vision, he is a
modern version of a white knight, the sole preserver of innocence.
 His other works include Nine Stories, Franny and Zooey, and Raise High the
Roof-Beam, Carpenters, a collection of stories from The New Yorker.
7. Jack Kerouac (1922-1969) was the son of an impoverished French-Canadian family; Jack Kerouac also questioned
the values of middle-class life.

19
 Kerouac's best-known novel, On the Road, describes "beatniks" wandering through America seeking an idealistic
dream of communal life and beauty.
 The Dharma Bums also focuses on peripatetic counterculture intellectuals and their infatuation with Zen
Buddhism.
 Kerouac also penned a book of poetry, Mexico City Blues, and volumes about his life with such beatniks as
experimental novelist William Burroughs and poet Allen Ginsberg.
8. John Barth (1930- ) is more interested in how a story is told than in the story itself. Barth entices his audience into
a carnival fun-house full of distorting mirrors that exaggerate some features while minimizing others. Many of his
earlier works were in fact existential.
 In Lost in the Funhouse, he collects14 stories that constantly refer to the processes of writing and reading.
Barth's intent is to alert the reader to the artificial nature of reading and writing, and to prevent him or her from
being drawn into the story as if it were real.
9. Norman Mailer (1923-2007) was a novelist, essayist, poet, playwright, screenwriter, and film director. He is
considered as an innovator of narrative nonfiction called New Journalism in Miami and the Siege of Chicago. He is
also famous for his compelling study about the execution of a condemned murderer in The Executioner's Song. In
the e1990s, he wrote such heavyweight novels as Ancient Evenings, set in the Egypt of antiquity, and Harlot's
Ghost, revolving around the U.S. Central Intelligence Agency.
10. Toni Morrison (1931- ) won the Nobel Prize for Literature in 1993 for her skillful rendition of complex identities
of black people in a universal manner. In her early work The Bluest Eye, a strong-willed young black girl tells the
story of Pecola Breedlove, who survives an abusive father. Pecola believes that her dark eyes have magically become
blue, and that they will make her lovable. Some of her novels include: Sula, Song of Solomon, Tar Baby, and
Beloved.
11. Alice Walker (1944- ) is an African-American who uses lyrical realism in her epistolary dialect novel The Color
Purple where she exposes social problems and racial issues.
 Walker's The Color Purple is the story of the love between two poor black sisters that survives a separation
over years, interwoven with the story of how, during that same period, the shy, ugly, and uneducated sister
discovers her inner strength through the support of a female friend. The theme of the support women give each
other recalls Maya Angelou's autobiography, I Know Why the Caged Bird Sings (1970), which celebrates the
mother-daughter connection, and the work of white feminists such as Adrienne Rich.

PHILIPPINE LITERATURE IN ENGLISH


1. What rhythmical device is used in this line from Latorena’s The Small Key? “…even the low square nipa house that
stood in unashamed relief against the gray green haze of grass and leaves.”
A. alliteration B. assonance C. consonance D. anaphora
2. What sense image is used in this line from Latorena’s The Small Key? “There were dark, newly plowed furrows where
in due time timorous seedlings would rise to sturdy stalks and golden grain, to a rippling yellow sea in the wind and
sun during harvest time.”
A. tactile B. olfactory C. visual D. auditory
3. “Soledad nodded. Her eyes followed her husband down the road, noting the fine set of his head and shoulders, the
ease of his stride. A strange ache rose in her throat.” Which emotion best describes Soledad’s feelings in this
excerpt from Latorena’s The Small Key?
A. fervent desire C. instinctive suspicion
B. discomfort from sore throat D. twinge of sadness
4. What figure of speech is used in this last paragraph from Latorena’s The Small Key? “How quiet and peaceful the day
was! A cow that had strayed by looked over her shoulder with a round vague inquiry and went on chewing her cud,
blissfully unaware of such things as a gnawing fear in the heart of a woman and a still smoldering resentment in a
man’s.”
A. irony B. simile C. personification D. metaphor
5. Alfredo Salazar undergoes epiphany at the end of Marquez-Benitez’ story. What does epiphany mean?
A. regret B. insight C. freedom D. confusion
6. What do ‘dead stars’ symbolize in Marquez-Benitez’ story?
A. the let-down feeling over the truth of a disillusionment
B. the dilemma between responsibility and freedom
C. the discovery of how life’s values change with time
D. the doubts and uncertainty that stand in a relationship.
7. “Things that are beautiful have a way of hurting. I destroy when I feel hurt.” What does Fabian’s thoughts reveal
about his character?
A. bitter B. goal-oriented C. persistent D. vindictive
8. “They dressed him in purple and linen, in myth and mystery, put him astride a black stallion, at the wheel of a blue
automobile.” How do the townspeople regard Mr. Reteche in Rotor’s Zita?
A. They find him difficult and confusing. C. They mocked him.
B. They were at awe with his presence. D. They respected and admired him.
9. “She did not have the courage to break into the wedding feast.” What did the ‘wedding feast’ stand for in the
relationship of Awiyao and Lumnay in Daguio’s “Wedding Dance”?
A. village tradition B. love for each other C. desire to have a child D. patriarchal society
10. “And far away in the middle of the fields a cow lowed softly in answer.” What sense image is used in this excerpt
from Arguilla’s How My Brother Leon Brought Home a Wife?
A. visual B. olfactory C. tactile D. auditory
11. “A little green snake slithered languidly into the tall grass a few yards from the kamansi tree.” What does the snake
symbolize in Love in the Cornhusks?
A. betrayal B. temptation C. distrust D. lust
12. “She was deep in the road before she became conscious of her shoes. In horror, she saw that they were coated with
thick, black clay. Gingerly she pulled off one shoe after the other with the hand still clutching the letter.” What does
the underlined word in this excerpt from Love in the Cornhusks mean?
A. disgustedly B. carefully C. hurriedly D. determined
13. What historical time is Nick Joaquin’s May Day Eve set?
A. Japanese occupation C. Spanish regime
B. American time D. Martial law era
14. What symbol is used to link the events of the three generations in May Day Eve?
20
A. guardia sereno B. candle C. mirror D. midnight
15. How long is the span of the story May Day Eve?
A. one month B. almost 50 years C. overnight D. 3 years
16. What does Magnificence refer to in Alfon’s story?
A. the girls’ innocence C. the father’s protectiveness
B. Vicente’s kindness and generosity D. the mother’s comforting presence
17. “There was nothing to fear, for the man was always so gentle, so kind.” What literary device is employed in this line
from Magnificence?
A. foreshadowing B. en medias res C. symbolism D. flashback
18. What is Cordero-Fernando’s view on the educational system in The Visitation of the Gods?
A. She hates the rotten side of the system. C. She is proud of the administrators.
B. She questions the role of educators. D. She upholds quality education.
19. What character of Miss Noel is revealed in this statement? “Sir, during the five years that I’ve taught I’ve done my
best to live to my ideals. Yet, I please nobody. It’s the same old narrow conformism and favor-curriying.”
A. militant B. stubborn C. idealistic D. pessimistic
20. “It was not quite five, and the bread was not yet ready.” What does the bread symbolize in N.V.M. Gonzales’ Bread
of Salt?
A. The boy’s unreciprocated love for Aida.
B. The embarrassment of the boy in the presence of Aida.
C. The boy’s dream of becoming a famous violinist.
D. The differences in the social status between Aida and the boy.
21. “I felt all ardor for her gone from me entirely.” What epiphany does the character experience in N.V.M. Gonzales’
Bread of Salt?
A. The boy did not like Aida’s aristocratic upbringing. C. He has found another girl to love.
B. His love for Aida was but an infatuation. D. Aida did not like him.
22. Which is an olfactory image?
A. The sun had sunk and down from the wooded sides of the Katayagan hills shadows were stealing into the fields.
B. He swallowed and brought up to his mouth more cud and the sound of his insides was like a drum.
C. The thick unpleasant smell of dangla bushes and cooling sun-heated earth mingled with the clean, sharp scent of
arrais roots exposed to the night air and of the hay inside the cart.
D. The wind whistled against my cheeks and the rattling of the wheels on the pebbly road echoed in my ears.
23. Which story shows the conflict of person vs. self?
A. The Visitation of the Gods C. May Day Eve
B. The Virgin D. Magnificence
24. Which of the following stories is in medias res?
A. Love in the Cornhusks C. Dead Stars
B. Wedding Dance D. The Small Key
25. Which story uses the first person narrator point of view?
A. Bread of Salt C. The Visitation of the Gods
B. How My Brother Leon Brought Home a Wife D. Harvest
26. Which story is a satire?
A. The Virgin C. The Visitation of the Gods
B. Wedding Dance D. Harvest
27. In which story does the main character experience epiphany?
A. Visitation of the Gods C. Bread of Salt
B. How My Brother Leon Brought Home a Wife D. The Small Key
28. Which among these objects is a phallic symbol in literature?
A. key B. cornhusks C. Waig D. pencil
29. Which story has a circular plot?
A. Dead Stars B. May Day Eve C. Magnificence D. Harvest
30. Which is not characteristic of a story of local color?
A. mores and traditions of a particular locality C. bias and prejudice of characters
B. description of the local setting D. politeness and address markers
31. The figure of speech implied in this line from Alfon’s Forever Witches: “MARING: Bastos! I know what you really
wanted to say. Pusa ka din!!”
A. metaphor B. pun C. personification D. hyperbole
32. What does Gonzalo mean by “a little longer” in this excerpt from Guerrero’s Three Rats?
Adrian: (Laughing): I expect to live a little longer, Gonzalo.
Gonzalo: A little longer is right. (Adrian’s face slowly begins to get red. He feels a giddiness in his head – he
presses his temples.)
A. He wants Adrian to apologize for his affair with Nita. C. Adrian is getting drunk and incoherent.
B. The cyanide in Adrian’s coffee is taking effect. D. His wife’s infidelity makes him suffer in silence.
33. “Gonzalo: I attach myself to nothing and to nobody.” What character trait does Gonzalo reveal in this excerpt from
Guerrero’s Three Rats?
A. shyness B. indifference C. pride D. boldness
34. Montano’s Sabina opens with this scene
A. novena prayers for the dead C. confrontation between Sabina & Mr. George
B. burial of Sabina’s mother, Maria D. Sabina lighting the lamp
35. “A bamboo flute is heard far away. Once more we hear the turtle crying.” What sense imagery is used in this
description of the scene?
A. visual B. auditory C. tactile D. olfactory
36. What is the tone of the father’s statement in this line from Florentino’s The Dancers? “ FATHER: (mimicking her).
She’s still a baby! I’m telling you, she’s old enough to have a
Baby!
A. bitter B. sarcastic C. thrilled D. hopeful
37. Which comic technique is used in Agana’s NewYorker in Tondo?
A. mistaken identity C. reversal of fortune
B. physical blunder D. ridiculous situation
38. This play is a satire
21
A. Forever Witches C. The Husband of Mrs. Cruz
B. The Dancers D. Sabina
39. Which of the following plays is a farce?
A. Forever Witches C. The Husband of Mrs. Cruz
B. The Dancers D. Sabina
40. This play is naturalistic in orientation
A. Sabina C. The Dancers
B. Forever Witches D. Three Rats
41. The rhyme scheme in Villa’s Sonnet 1
A. quatrain B. couplet C. octave D. sestet
42. What characteristic of poetry is referred to by the line “It must be slender as a bell” in Villa’s
Sonnet 1?
A. rhyme & rhythm C. form & structure
B. figurative language D. imagery
43. What emotion is expressed in these lines from Manalang-Gloria’s Poems? “The madcap
inspirations, bent/On flinging stars about,/Contrive to break away before/I know that they are
out”
A. confusion B. excitement C. awe D. fear
44. What figure of speech is used in this line from Alvero’s poem? “Could Gods refuse/Such
tempting wine?”
A. apostrophe B. rhetorical question C. allusion D. metaphor
45. This line is an example of personification
A. And hold secret a bird’s flowering C. And so while thoughts went to and fro
B. Seeking a truer heaven in/The loved deep D. to teach the trees all that I could

AFRO-ASIAN LITERATURE
1. According to this religion human beings are bound to the wheel of life which is a continual cycle of birth, death, and
suffering.
A. Hinduism B. Buddhism C. Shintoism D. Taoism
2. __________ is a collection of sacred hymns in archaic Sanskrit which exalt the deities who personify various natural
and cosmic phenomena.
A. Dhammapada B. Upanishads C. Bhagavad Gita D. Rigveda
3. This is a story of a learned Brahman named Vishnusarman who used animal fables to instruct the three dull-witted
sons of a king.
A. Panchatantra C. Gitanjali
B. The Little Clay Cart D. On Learning to be an Indian
4. __________ dominates every scene in a Sanskrit drama and allows the audience to take part in the play and be one
with the characters.
A. artha B. rasa C. kama D. moksha
5. What is the rhythmical development of this excerpt from the Rigveda, ‘The Hymn of Man?’

When they divided Purusa, how many portions did them make?
What do they call his mouth, his arms? What do they call his thighs and feet?
The Brahman was his mouth, of both arms was Rajanya made.
His thighs became Vaisya, from his feet the Sudra was produced.

A. free verse B. quatrain C. couplet D. octave

Read the poem by Ping Hsin then answer questions 6 –7 Time is a Pair of Scissors
6. What figure of speech is used in the title? Ping Hsin
A. simile C. personification And life, a bolt of brocade
B. metaphor D. hyperbole Section by section the brocade is cut;
7. What is the central idea of the poem? When the last section is done
A. cruelty of time C. destruction of beauty The scraps are committed to a bonfire.
B. impermanence of life D. beauty of nature
Time is an iron whip,
8. All of these are Chinese genres in poetry except _______. And life, a tree full of blossoms.
A. li sao C. shih One by one the flowers are lashed off;
B. chueh-chu D. renga When the last on is gone,
9. Which is not characteristic of Chinese theater? The fallen petals are trampled into the dirt and sand.
A. follows the unities of time, place, and action
B. conveys an ethical lesson in the guise of art in order to impress a moral truth
C. a total theater with singing, recitation of verses, acrobats, and dancing
D. there are two types of speeches – the dialogue and the monologue
10. China’s most famous teacher, philosopher, and political theorist, whose ideas have influence all civilization of East
Asia.
A. Confucious B. Lao-tzu C. Li Po D. Tu Fu
11. A farce normally performed between the Nō tragedies
A. kabuki B. kyogen C. Jorori D. bunrako
12. Yukio Mishima’s four-part epic including Spring Snow, Runaway Horses, The Temple of Dawn, and The Decay
of the Angel is known as __________.
A. The House of Sleeping Beauties C. Snow Country
B. The Sea of Fertility D. The Wild Geese
13. He is one of the most widely translated of all Japanese writers, and a number of his stories have been made into
films such as Rashomon
A. Yasunari Kawabata C. Oe Kenzaburo
B. Junichiro Tanizaki D. Ryunosuke Akutagawa
14. What is the atmosphere created by the image of nature when linked with the image of the woman?

22
A. romance
Blossoms on the pear; B. boredom
And a woman in the moonlight C. weariness
Reads a letter there … (Buson) D. excitement

Read the excerpt below from Chinua Achebe’s The Voter then answer questions 15 –16.
We have a Minister from our village, one of our own sons. He said to a group
of elders in the house of Ogbuefi Exenwa, a man of high traditional title,
“What honour can a village have? Do you ever stop to ask yourself why we
5
should be single our of this honour? I will tell: it is because we are favoured by
the leaders of PAP. Whether we cast our paper for Marcus or not PAP will
continue to rule. Think of the pipe-borne water they have promised us …
Besides Roof and his assistant, there were five elders in the room. An old
hurricane with a cracked sooty glass chimney gave out yellowish light in their

10
midst. The elders sat on very low stools. On the floor, directly in front of them,
lay two shilling pieces. Outside the moon kept a straight face. “We believe
every word you say to be true,” said Ezenwa. “We shall every one of us drop
his paper for Marcus. Who would leave an ozo feast and go to a poor ritual
mean? Tell Marcus he has our papers, and our wives papers, too. But what we
15
do say is that two shillings is shameful.” He brought the lamp close and tilted
it at the moment before him as if to make sure he had not mistaken its value.
“Yes, two shillings, it is shameful. If Marcus were a poor man which our
ancestors forbid I should be the first to give him my paper free, as I did before.
But today Marcus is a great man. We did not ask him for money yesterday; we
20
have climbed the iroko tree today and would be foolish not to take down all the
firewood we need.”

15. The ‘iroko’ tree mentioned in line 20 symbolizes __________.


A. responsibilities B. privileges C. opportunities D. resources
16. What ills in society are being condemned in this satirical story?
A. unqualified yet popular candidates C. politician’s unkept promises
B. opportunities during election campaign D. vote-buying during elections
17. __________ is the movement organized by African writers who wished to reunite black people to their own history,
traditions, and languages, to the culture which truly expresses their soul.
A. Tigritude B. Apartheid C. Negritude D. Orature
18. The study of Asian literature cannot be dissociated from all of these except one.
A. history B. religion C. philosophy D. politics
19. The world’s first known novel, The Tale of Genji was written by __________.
A. Lady Murasaki B. Wu Chengan C. Sei Shōnagon D. Valmiki
20. The religion of this country is based on the perception of life as a process of continual change in which opposing
forces, such as heaven and earth or light and dark, balance one another.
A. India B. China C. Japan D. Africa

ANSWER KEY
Philippine Literature in English
1. A 9. A 17. A 25. A 33. B 41. B
2. C 10. D 18. A 26. C 34. A 42. C
3. D 11. B 19. C 27. C 35. B 43. B
4. A 12. B 20. D 28. D 36. B 44. B
5. B 13. C 21. B 29. B 37. D 45. C
6. A 14. C 22. C 30. C 38. C
7. A 15. B 23. B 31. B 39. A
8. D 16. D 24. A 32. B 40. C

Afro-Asian Literature
1. B 5. C 9. A 13. D 17. C
2. D 6. B 10. A 14. A 18. C
3. A 7. A 11. B 15. C 19. A
4. B 8. D 12. B 16. D 20. B

BASIC SENTENCE PATTERNS


1. N be Adj where the adjective is a subject complement, in particular a predicate adjective. The adjective refers back
to the subject. The copula verb be means “may be described as.”
Roses are sweet.

2. N be UW (= uninflected word),
where the uninflected word is an ADVERBIAL such as here, there, up, down, in, out, inside, upstairs,
downstairs, on, off, now, then, yesterday, and tomorrow. Be has the meaning of “be located” or “occur.”
The meeting was yesterday.

3. N1 be N1 where the superscript means that the two nouns have the same referent. The second noun following the
be verb is also a subject complement, in particular a PREDICATE NOUN or PREDICATE NOMINATIVE.
Her neighbor is my cousin.

4. N InV (= intransitive verb)


where the intransitive verb does not require an object. The verb being self-sufficient can stand alone with
its subject.
Glasses break.
23
5. N1 TrV (= transitive V) N2
where N2 does not have the same referent as N 1. N2 is called the direct object of the verb, “the receiver of
the action.”
The girl buys yellow roses.

6. N1 TrV N2 N3
where the superscripts 1, 2, and 3 indicate that each noun has a different referent.
Mother gave a gift to the orphan. (usually reads as Mother gave the orphan a gift.)

Two noun objects occur after the verb. Still N2 is the direct object and N3 is the indirect object. If we omit the last noun,
the pattern is identical to that in item 5. Note that the indirect object is preceded by the preposition to (sometimes for or
of). If the two objects are inverted, the preposition disappears.
He made a toy house for her.
He made her a toy house.

The teacher asked a question of her.


The teacher asked her a question.

7. } N2
} Adj
} Pronoun
N1 TrV N2 } Adv (of place), uninflected
} Verb, present participle
} Verb, past participle

There are a choice of different forms in sentence final position. These are illustrated as follows:
The class voted Henry secretary.
The principal found the gardener efficient.
We considered the writer you.
The teacher directed them outside.
She saw them praying.
I imagine my father overworked.

The most common illustration of this sentence pattern is one with the occurrence of a final N 2.

NOUNS
Nouns can be recognized by means of the following characteristics:
1. They are names of entities -- a person, place, thing, of idea.
2. They have two inflections, the plural {-es} and the possessive (sometimes called the genitive) {-‘s}. Both inflections
have various allomorphs
/əz/ appears after morphs ending in sibilants or affricates / s, z, š, ž, č j /
/s/ appears after morphs ending in voiceless consonants / p, t, k, f, Ɵ /,
except the sibilants and affricate / s, š, č /
/z/ appears after morphs ending in vowels and voiced consonants / b, d, g, v,
ð, m, n, ŋ, l, r. y, w /, except the sibilants and the affricate / z, ž, j /
3. They may be marked by noun-forming derivational suffixes added to bases or stems, usually belong to other parts of
speech, e.g.
 added to verbs
{-age} breakage
{-ee} employee
 added to adjectives
{-ity} facility
{-ness} happiness
 added to other nouns
{-cy} advocacy
{-ian} librarian
{-ship} friendship

4. They fill certain characteristic positions in relation to other parts of speech in phrases and sentences.
 just before a verb
Red roses bloom in my garden.
 after determiners such as articles, demonstratives, and possessive adjectives, e.g. the examination, these
reviewees, my handouts
5. Unlike other languages, gender is not an important feature of English grammar. Gender is only marked in certain
pairs of nouns, e.g. waiter/waitress, host/hostess
6. Certain superfixes occasionally identify nouns from other parts of speech as in: récòrd and rècórd. These two words
are morphemically alike; however, we identify the stress pattern / ˊ ˋ / as a noun.
7. Nouns can serve as heads in a noun phrase. As heads they may be preceded by one or more single-word modifiers
and followed by a phrasal or clausal modifier or both
the small study table in my room which my father bought

Functions of Nouns
subject of verbs Several items have ambiguous stems.
direct objects of verbs They administered the test.
indirect objects of verbs The lecturer provided the participants handouts.
subject noun predicates/ We are LET reviewers.
predicate nouns
24
object noun predicates/ The reviewees chose him their representative.
object complements
objects of prepositions in our review class
appositives The LET, a professional examination, is conducted every year.
vocatives/nouns of address Anne, how did you find the exam?

Noun Types
1. common: nouns referring to a kind of person, thing, or idea
 Count nouns, which take the plural inflection
 Mass/noncount nouns, which don’t take the plural inflection
2. proper nouns: names for unique individuals or places
3. collective nouns: able to take either singular or plural verbs forms, depending on the interpretation given to the
noun, i.e, whether it is seen as a unit or as a collection of individuals
The team has won all its games.
The team have won awards in their respective events.

ARTICLES
Articles are a subclass of determiners, which are noun-marking words. They usually come before the nouns they modify.

a/an (indefinite articles) the (definite article) no/zero article


Only before singular (sg) Before uncountable/mass nouns (UNs) Identifies certain indefinite
countable nouns (CNs) and countable plural (pl) nouns meanings of nouns
Before an unindentified sg CN, Reference backward to a N already To refer to all members of a
1 one example of its class mentioned class
a chair (furniture) A dog . . . and here is the dog now. 0Dogs are domestic animals.

Reference forward to an identification To distinguish one class from


Before an unindentified sg CN
soon to be made, often by modifiers another
2 that is representative of its class
following a noun 0Men, not women, are
a dog (a domestic animal)
The history of his town protectors.

Before superlatives and before ordinal To refer to an indefinite number


Before a predicate N after a be numbers, except ordinal numbers used but not necessarily to all
3 verb if no determiner is used alone (first in her batch) members of a class.
. . . is a good neighbor The best cake I have ever seen 0Seedlings are beginning to
The first person to fly in space sprout. (many)

With UNs to mean mean a kind


Content know to both writer and reader
of, or with kind of, or certain With plural nouns after be.
4 The chapel in the village (only one
a smile His brothers are 0engineers.
chapel)
an insight
Identification of a class, especially in a
With institutions and practices
generalization,
Before few and little to mean felt to be unique
5 followed by a noun, or an adjective
some but not many 0Offices open at 7 o’clock.
The youth is the hope of the future.
0Dinner is usually late.
the physically challenged

When using a proper noun to


Beginning of a phrase containing an
indicate the characteristics of
appositive With set phrases, usually pairs
6 the person named
Interpret this item, the one with an 0Heaven and hell
She is a Sister Teresa. (a saintly
illustration
person)

To name “a certain person


With prepositional phrases
7 whose name is.”
0At rest, in danger, on time
A Mr. Alba came to see you.

With nouns used in headlines in


newspapers, captions in books,
8 signs, labels and the like
0MURDERER ESCAPED
BEWARE OF 0DOG
With common nouns used as
For a family name in the plural terms of address and therefore
9
The Basas have arrived. capitalized.
We are ready to go, 0Mother.
To distinguish people who have the
same name
10 The Jessica Reyes who joined the
beauty pageant is not the Jessica Reyes
who is my cousin.

25
When the article is part of a
geographical name
11 The Philippines
The United States
The Red Sea

When the article is accepted as part of


any kind of proper name
The Philippine Star
12 The Princess of Negros
The Hilton
The University of St. Tomas
The United Nations

PRONOUNS
Most pronouns stand for, refer to, or replaces a noun or a noun phrase within a text; hence, they occupy the same
position as a noun or noun phrase does. The word or words that a pronoun stands for are its antecedent or reference.
My brother holds dual citizenship. He is not only a Filipino but also a Canadian citizen.
I and me stand for the speaker or writer.
I am a Filipino, but I am living in Australia now.

Pronouns can also be a direct reference to an outside situation (e.g., “What is that?” in response to a sound or noise).

Kinds of Pronouns
There are many different kinds of pronouns: subject, object, possessive, reflexive, demonstrative and others. The forms
within each category are distinguished by number (singular/plural), person (first/second/third) gender
(masculine/feminine/neuter), and in the case of demonstratives, by number and proximity.

Personal and Related Pronouns


Person/
Personal Possessive
Number
Reflexive/
Subject Object Noun Determiner/
Intensive
Singular Form Form replacement Adjective
+I I me mine my myself
+ II you you yours your yourself
+ III
Masculine he him his his himself
Feminine she her hers her herself
Neuter it it - its itself
Plural
+I we us ours our ourselves
+ II you you yours your yourselves
+ III they them theirs their themselves

Things to remember:
1. Animals closely related to people can be referred to by he, him, and his or she, her, and hers.
The dog loves his/her/its master.

2. Use it and its to refer to inanimate objects except ships, which are always referred as she.
3. Countries and schools are sometimes referred matter are sometimes referred to by she or her.
4. Traditionally, the pronouns he, him, and his have been used for mixed groups or groups in which the sex is
unknown. Many people now object to this use, so they use both the masculine and feminine forms or the plural
forms to avoid the problem.
Everybody submitted his or her assignment. (awkward)
All the students submitted their assignments. (acceptable)

5. If I, me, my or mine or their plural counterparts are part of a pair or a series, put them last.
The teacher confiscated his toy and mine, too.
Father helped Tony with his project, and he will help my sister and me with ours tomorrow.

Reflexive Pronouns
1. Use the reflexive pronoun as the object of the verb form or preposition to refer to the subject of the sentence.
The baby is able to feed itself.
Luis cut himself with a razor blade.
2. The phrase by + self or its emphatic form all by + self means alone or without any help.
I crossed the river (all) by myself.

Intensive Pronouns
The intensive form occurs directly after the word it modifies or at the end of the clause.
The mayor herself distributed the relief goods.
The mayor distributed the relief goods herself.

Reciprocal Pronouns
1. The reciprocal pronoun forms are each other and one another. They means that each part of the subject did the
action and also received the action.
2. They must be objects of verb forms or objects of prepositions.
3. Some prefer to use each other for two people or things and one another for more than two.
The two finalists congratulated each other for making it to the top.l
26
The class members prepared surprise gifts for one another during the Christmas party.

Demonstrative Pronouns
1. Demonstrative pronouns occur alone. They do not precede nouns.
This is my favorite movie.
2. Demonstrative pronouns can show distance or contrast not connected with distance.
This is mine; that is yours over there. (distance)
Which ones do you prefer, these or those? (contrast)

Indefinite Pronouns
none another
anyone everyone someone
Personal no one other ones
anybody everybody somebody
nobody others
another
everything nothing
NonPersonal anything something other ones
every one none
others

Use singular verbs with compound pronouns and use singular pronouns to refer to them in formal writing.
Formal: Nobody brought his book today.
Informal: Nobody brought their books today.

Question Interrogative pronouns


Who, whom, whose, which, and what can begin questions.
1. Use who, whom, whose and which to refer to persons..
2. Use what and which to refer to things and events.
3. In formal writing, use who for the subject of a clause and whom for the object of the verb or preposition.

Relative pronouns
1. Relative pronouns (sometimes called clause markers) introduce dependent clauses (also called relative clauses).
2. Relative pronouns used in adjective clauses are who, whom, whose, which and that.
3. Who, whom, and whose are used for persons while which is used for non-persons.
The guest who came to dinner is the governor.
The book which I bought is a best seller.
4. That is a neutral form. It can be marked +human or –human. In other words, it can be a substitute for both who
(+human) or which (-human).
The guest who/that came to dinner is the governor.
The book which/that I bought is a best seller.
5. In informal writing, whom is optional; in formal writing, whom must be used
Nora is the girl you saw in the party last night. (informal)
Nora is the girl whom you saw in the party last night. (formal)
6. That, which and whom are the only relative pronouns that can be left out.
The instrumental music (that) I like to hear often is that of Zamfir.
The house pests (which) I hate to see are the rodents and the cockroach.
7. Who, whom, and whose can be used in both essential and nonessential clauses.
8. That instead of which is used only in essential or restrictive clauses, so do not put commas around clauses beginning
with that.
*The poster, that won first prize, pleased both the judges and the viewers.
9. Use which in nonessential or nonrestrictive clauses. Separate nonessential clauses from the rest of the sentence
by commas.
Our car, which has been running for three days, should be brought to the machine shop for check-up.
10. Relative pronouns used in noun clauses are that, what, whatever, whoever,
whomever, and whichever.
Whatever you offer will be appreciated. (noun clause as subject)
He will befriend whoever he gets acquainted with. (noun clause as direct object)
11. Look at the antecedent of who, that or which when used as subject to decide
whether the verb following should be singular or plural.
The painting which is exhibited is the painter’s masterpiece.
The farmers who own orchards earn much from their harvest.

PREPOSITIONS
Prepositions are notoriously difficult for ESL learners for several reasons.
1. Several English prepositions are realized as a single form in the learner’s first language.
Pumunta kami sa palengke. (We went to the market).
Lumangoy kami sa ilog. (We swam in the river.)
Sa kalye ang gulo. (The commotion occurred on the street.)
Antayin mo ako sa kanto. (Meet me at the street corner.)
2. The English preposition is not necessarily realized by a single word. There are complex forms like because of and
in spite of or coalesced forms like into and onto.
3. Certain prepositions co-occur with verbs, adjectives, and nouns to form clusters.
to substitute for to be afraid of
in favor of awareness of
4. English prepositions are polysemous. They bear varied meanings.
Throw the at the wall. (space)
It rains at night. (time)
Water freezes at 00C. (degree)
She’s good at dancing. (idiomatic)

27
Meanings of Prepositions
1. Many prepositions prototypically deal with locating objects in space involving two or more entities. One entity is
for foregrounding, while the other serves as background. The former is the figure and the latter is the landmark.
In
Throw the ball at the wall.
figure landmark
2. Note the following figure
at on in
by ↕ ↕ ↕ through
with about
under over
from off out of

Adapted - Dirven 1993


 At, on, and in are the basic and most general place prepositions. At denotes place as a point of reference, on
denotes physical contact between the figure and landmark, and in denotes the enclosure of the trajector in
the landmark.
They met at the main gate.
Put the box on the table.
The ball is in the box.

 From, off, and out of are source prepositions involving the notion of separation from place. From denotes
separation from a point of orientation, off denotes separation from contact with line or surface, and out of,
separation from inside a landmark.
We walked from the gate to the waiting shed.
The box fell off the table.
Take the ball out of the box.

 By and with are proximity prepositions, which locate the figure in relation to a point of orientation marked by
the preposition at. By denotes the idea of “connection” while with denotes both a point of orientation and the
idea of connection. In its spatial sense, with can occur only with animate nouns as landmark.
He stood by me in all throughout the campaign.
He rides with me to our place of work.

 Through and about require the landmark to the seen as a surface or a volume and are there positioned in the
diagram above next to in. Through structures space as a tunnel or channel. About denotes spatial movement
in any direction.
Move the other side of the mountain through the tunnel.
He walked briskly about the yard for his morning exercise.

 Under and over are vertical space preposition. Under denotes a figure at a lower point than the landmark.
Over denotes a figure that is at a higher point than the landmark.
Don’t keep your shoes under the table.
We watched the game over the fence.

Selected Meanings and Uses of Common Prepositions


1. at We left at 2:00 pm. (exact)
2. about We left about 2:00 pm. (approximate)
3. against to lean against the wall (contact)
4. around around 2:00 (approximate)
5. by bed by the window (nearness)
by 2:00 (no later than)
6. from paper is made from wood (source)
7. of a quarter of ten (before)
8. on on the wall (contact)
on Sunday, on November 8th (day, date)
on radio, TV (communication)
a round-table discussion on language policy (concerning)
9. over over the weekend (spanning time)
over the radio, TV (communication)
10. through through the forest (penetrate)
through thick and thin (endurance)
11. to work from 8 to 5 (until)
a quarter to 11:00 (before)
He is honest to such extent. (degree)
12. under in under an hour (less than)
under stress (condition)
13. with He grew smarter with the years. (together)
rank with the best (equal standing or ability)
delivered his speech with ease (manner)

Variations in Use of Prepositions


1. spatial proximity a house near/by the lake
2. time/degree approximation cost about/around Php1,500.00
3. telling time a quarter of/to ten
a quarter after/past ten
4. location along something linear the houses on/along the river

28
5. in a time period It occurred in/during 1901.
6. temporal termination studied from 8 until/till/to 5
7. location lower than something below/beneath/under/underneath the bed
8. location higher than something above/over the piano
9. location in/at the rear of something behind/in back of the cabinet
10. location adjacent next to/beside the cave

VERBS
A verb can be recognized by means of the following characteristics:
 Denotes an action (e.g., read) or a state of being (e.g. know). Action verbs are dynamic. State of being verbs (or
stative verbs) include the copula or linking verbs, e.g. the be-verbs, remain, appear, and become.
 Has four inflections
- {-s} of third person singular present tense verbs
- {-ed} of simple past tense verbs
- {-en} of the past participle
- {-ing} of the present participle
The third person singular –s has the same allomorphs as the noun plural and the noun possessive.
The –ed past tense inflection has three allomorphs:
- / əd / after morphs which end in / t / or / d / as in planted, raided
- / t / after morphs that end in voiceless sounds except / t / as in
brushed, jumped, walked
- / d / after morphs which end in voiced sounds except / d / as in cleaned, grabbed, agreed
 Follows a subject noun and may be followed in turn by adjectives
}______ eager [to enhance their knowledge].
The reviewees }______ seriously.
}______ their handouts.
 May fall under one more or more of these types
 intransitive verbs, which does not take an object (direct)
Flowers bloom.
 transitive verbs, which require an object (direct)
Flowers need water and sunlight.
 ditransitive verbs, which take two objects (direct and indirect)
Alex gave his girlfriend three red roses.
 linking/copula verbs, where what follows the verb relates back to the subject (subject complement -- a
predicate noun or a predicate adjective)
Roses are lovely Valentine’s Day gifts.
Roses are sweet.
 complex transitive verbs, where what follows the object (direct) relates to the object
They chose Niña, muse of the team.
 prepositional verbs, which requires a preposition phrase to be complete
We looked at the pictures taken during our graduation

 Have tense and aspect qualities. Tense and aspect have to do with form. TENSE is “the grammatical marking on
verbs that usually indicates time reference relative to either the time of speaking or the time at which some other
situation was in force” (Jacobs 1995). Time reference has to do with meaning. Events and situations are located
in time, perhaps to our speaking about them, perhaps while we are speaking about them, or perhaps at some
later time. English has three tenses – present, past, and future. The present and the past tenses have inflectional
markings, while the future is marked by the inclusion of the modals will or shall. Simply put, tense is a set of
verb forms that indicate a particular point in time or period of time in the past, present, or future.

ASPECT is a general name given to verb forms used to signify certain ways in which an event is viewed or
experienced. Aspect can view an event as completed whole (simple), or whether or not it has occurred earlier
(perfect aspect) or is still in progress (progress).
Noel has attended the review classes. (perfect)
Now he is studying for the LET exam. (progressive)
The tenses in combination with aspects make up the following 12 tense-aspect categories. These make up the
traditional 12 tenses.

Simple Perfect Progressive Perfect Progressive


0 have + -en be + -ing have + -en be +-ing
dream/dreams has/have dreamed am/is/are dreaming has/have been dreaming
Present
eat/eats has/have eaten am/is/are eating has/have been eating
dreamed had dreamed was/were dreaming had been dreaming
Past
ate had eaten was/were eating had been eating
will/shall will/shall have will/shall be will/shall have been
dream dream dreaming dreaming
Future
will/shall have been
will/shall eat will/shall have eaten will/shall be eating
eating

Sometimes, if we want to draw attention to the time of the action, we use an ADJUNCT OF TIME, which can be an
adverb, a noun group, or a prepositional phrase, e.g..:
She’s coming tomorrow. (adverb)
Results of the examination were released last week. (noun group)
He will feel relieved after the exam. (prepositional phrase)

Meanings and Common Uses of Verb Tenses


Simple Aspect: complete wholes; unchanging
1. Simple present: the present in general

29
a. To talk about our thoughts and feelings at the present moment or about our immediate reactions to
something
I’m terribly busy.
He looks excited.
b. To talk about a settled state of affairs which includes the present moment
He lives in Sagada now.
Our teacher is very competent and considerate. We like her very much.
c. To say something is always or generally true
There are 24 hours in a day.
The earth revolves around its axis.
d. To talk about something that a particular person or thing does regularly or habitually.
I get up early to take a bath.
Every Sunday, I attend church services.
e. To discuss what happens in a book, play or film
In the movie, he plays the character of Juan Tamad.
In those early chapters, he keeps himself isolated to other people in the village.
f. To describe an event such as a sports match or a ceremony at the time it is happening as radio and TV
commentators do
Doods takes the ball, then passes quickly to Alfie. Alfie turns, shoots, and scores two points.
2. Simple past: Stating a definite time in the past
An adjunct of time or other time expression is necessary o specify the particular time in the past we are referring
to.
a. To say that an event occurred or that something was the case at a particular time in the past.
The university officials flew into Jakarta last week to sign a memorandum of agreement with a
sister school.
b. To say that a situation existed over a period of time in the past.
He lived in his ancestral home in the countryside during his last years.
c. To talk about an activity that took place regularly or repeatedly in the past, but which no longer occurs
We swam in the river a great deal in my childhood.
3. Simple future: An expression of what we think might happen or what we intend to happen
a. To say that something is planned to happen, or that we think it is likely to happen in the future
What do you think Ella will do to fix it?
b. To talk about general truths and to say what can be expected to happen if a particular situation arises
An attack of dengue fever can keep a man off work for a few days. He will earn nothing and he
have trouble paying his hospital bills.

Perfect Aspect: prior


1. Present Perfect: the past in relation to the present
We cannot use adjuncts or expressions which place the action at a definite time in the past. But we can use
adjuncts of duration, e.g. forever, always.
*I have watched it the other day.
I ate raw vegetables, which I always avoided, and there was no other choice.
To mention something that happened in the past but we do not want to state a specific time.
I have read the book several times.
2. Past Perfect: Events before a particular time in the past
To talk about a past event or situation that occurred before a particular time in the past
By noon, students had gathered at the quadrangle with their placards.
3. Future Perfect
To refer to something that has not happened yet, but will happen before a particular time in the future.
By the time he graduates, his parents will already have left for New Zealand

Progressive Aspect: incomplete action; changing


1. Present progressive: Accent on the present
a. To talk about something that is happening at the moment we are speaking
I’m already feeling bored and hungry.
b. To emphasize the present moment or to indicate that a situation is temporary
She’s spending the summer in her hometown.
c. To indicate changes, trends, developments, and progress
He’s performance in class is improving.
d. To talk about a habitual action that takes place regularly, especially one which is new or temporary
She’s spending a lot on clothes these days.
2. Past Progressive: accent on the past
a. To talk about continued states or repeated actions which occurred in the past
His body was trembling; his fever was rising.
b. To contrast a situation with an event which happened just after that situation existed. We use the past
continuous to describe the first event and the simple past to describe the event which occurred after it.
We were standing at the main gate waiting to welcome the guest speaker. He arrived 20 minutes
later.
3. Future progressive
a. To say something will surely happen because arrangements have been made
They will be sending their students regularly to the University for English proficiency
enhancement.
b. To emphasize the duration of a recent event
She’s been crying bitterly.

Perfect-Progressive Aspect
1. Present perfect progressive
a. To talk about an activity or situation that started at some time in the past, continued, and is still
happening now.
30
The economy has been declining in many parts of the world.
2. Past perfect progressive
a. To emphasize the recentness and duration of a continuous activity which took place before a particular
time in the past.
The old woman had been living alone in that dilapidated house.
b. To say that something was expected, wished for, or intended before a particular time in the past.
I had been expecting a phenomenal rise in his political career.
3. Future perfect progressive
a. To emphasize the duration to an event at a specific time in the future
By January 2011, she will have been serving this university for 38 years.

Other concepts related to verbs


1. Verb phrase/verb complex: consists of an auxiliary + a main verb, e.g., must work, have been reading, will be
informed. The word or words in boldface are the auxiliary or help verbs.
2. Auxiliary/helping verbs
A. Modal auxiliaries and their related phrasal forms

True Modals Phrasal Modals


can, could be*able to
will, shall be going to, be about to
must have* to, have got to
should, ought to be to, be supposed
would (= past habit) used to
may, might be allowed to, be permitted to

 Non-modal auxiliaries : be, do, and have verbs


Of all the auxiliaries, only the non-modals can change form.
Distinguishing characteristics between true modals and phrasal modals

True Modals Phrasal Modals


Do not inflect, i.e., the forms Inflect like other ordinary verbs
1 remain unchanged am/is/are/was/were able to
can pass pass
Lacks tense and a resultant lack Subject-agreement rule applies
of subject-verb agreement except the form used to
2
We can pass the LET. We are able to pass the LET.
He can pass the LET. He is able to pass the LET.
Do not require an infinitive
Requires an infinitive marker to
marker to to precede the main
3 to precede the main verb
verb
has/have/had to study hard
must study hard

3. Operators/operator verbs
 The operator is a verb that has three main functions: 1) It precedes the negative and combines with it
when the negative is contracted to –n’t; 2) It is the verb that moves around the subject to the sentence
initial position in yes-no questions; and 3) It is also the verb that appears in the tag phrases of
interrogative sentences or tag questions.
My father will not approve your marriage proposal.
My father won’t approve your marriage proposal.
Will your father approve my marriage proposal?
Will your father not approve my marriage proposal?
Won’t your father approve my marriage proposal?
Your father will approve my marriage proposal, won’t he?
 When a clause contains no verb eligible to be an operator, do is introduced.
He attends the graduation ball tonight.
He does attend the graduation ball tonight.
He does not attend the graduation ball tonight.
Does he attend the graduation ball tonight?
He attends the graduation ball tonight, doesn’t he?
 If there are two auxiliary or more auxiliary verbs present in the verb phrase, the first auxiliary serves as
the operator.
He has been reading the Obama autobiography.
He has not been reading the Obama autobiography.
*He has been not reading the Obama autobiography.
Has he been reading the Obama autobiography.
He has been reading the Obama autobiography, hasn’t he?

SUBJECT-VERB AGREEMENT/CONCORD RULES WHICH OFTEN CAUSE ERRORS


1. Collective nouns may take either a singular or plural verb inflection depending on the meaning.
 Conceived of a one entity – takes singular verb
Our school team has won its games.
 Conceived of as more than one entity or refers to individual membership – takes plural verb
Our school team have won all their games.
2. Some common and proper nouns ending in –s, including –ics nouns and certain diseases are always conceived as
single entity – take a plural verb.
The recent news is exciting.
Mathematics is repelling to many students.
Measles is a contagious disease.

31
The United States is still a powerful country.
3. Titles of works even when plural in form are conceived of as single entitles.
The Ten Commandments is a beautiful movie.
The Syntax Files is good reading for those in linguistics.
The song Greenfields brings nostalgia to people of my generation.
4. Nouns occurring in sets of two take the singular when the noun pair is present but take the plural when pair is
absent.
That pair of Lee jeans is expensive.
My glasses are missing.
5. Fractions and percentages takes a singular verb inflection when modifying a noncount noun and the plural verb when
they modify a plural noun. Either a singular or plural verb inflection may be used when they modify a collective noun,
depending on the speaker’s meaning.
More than half of the cake is eaten.
Twenty percent of the students are not joining the field trip.
One-fourth of the audience is/are teachers.
6. A number normally takes the plural. The number takes the singular.
A number of parents are coming for the meeting.
The number of signatories is substantial to merit approval of the motion.
7. When we use a number and a plural noun to talk about two or more things, we usually use a plural verb. We use a
singular verb with ‘one’.
Seven days make up a week.
One solid evidence is enough to prove his dishonesty.
8. When we are talking about an amount of money or time, or a distance, speed, or weight, we usually use a number, a
plural noun, and a singular verb.
Five hundred dollars is a lot of money.
Three years is a long time to wait for a family member from abroad to come home.
Eighty kilometers per hour of travel is quite risky on slippery roads.
Seventy-five pounds is all she weighs now.
9. Arithmetic operations take the singular because they are perceived as reflecting a single numerical entity on both
side of the equation or equal sign.
Two plus two is/equals four.
10. The quantifiers a lot (of), lots of, plenty of take a singular verb if the subject noun is noncount by plural verb if the
subject head noun is plural.
A lot of sound views were advanced during the discussion.
A lot of nonsense is evident from uninterested participants.
11. Traditional grammar states that when used as a subject, none (meaning not one) is always singular regardless of
what follows in a prepositional phrase.
None of the boys joins the mountaineering group.
None of rice is eaten at all.
12. Traditional grammar maintains that the antecedent of the relative pronoun is the noun before.
Alice is one of the graduate students who have finished her master’s degree in a short period of time.
13. For correlatives either . . . or and neither . . . nor, traditional grammar argues for a proximity rule, i.e., subject-verb
agreement should occur with the subject noun nearest to the verb.
Either my friend or my classmates are expected to help me with my project.
Neither my classmates nor my friend volunteers to lend support.
14. A singular noun or pronoun should take a singular verb inflection regardless of what else occurs between the subject
and the verb.
Jimmy, along with his co-teachers, conducts a cleanliness campaign in the barangay.

VOICE
VOICE pertains to who or what serves the subject in a clause. In the active voice, the subject of a clause is most often
the agent, or doer, of some action. In the passive voice, the subject of a clause is the receiver or undergoer of the
action. The passive “defocuses” the agent. (Shibitani 1985 in Celce Murcia and Larsen-Freeman 2001)
The lifeguard saved the child. (active)
The child was saved [by the lifeguard]. (passive)

The passive voice is more limited than the active in that it requires only the transitive verbs – verbs that take direct
objects.
The passive morphology is be . . . –en, i.e., a form of the be verb + the past participle. Usually in passive sentences the
agent is not mentioned at all, referred to as the agentless passive. If the agent is mentioned (= agented passive), it
appears in a prepositional phrase marked by the preposition by.

Some passive sentences have no active counterparts.


Justin was born in Canada.

Advantages of the Active Voice


1. An active clause can give more information in fewer words.
2. A active verb makes writing livelier and more vivid.

Uses of the Passive Voice


1. A passive construction emphasizes the result in an impersonal style. This use is sometimes desirable in scientific
and technical writing.
A new strain of malaria was discovered.
2. A passive verb emphasizes a victim or the result of a disaster.
Active: The child broke the antique vase.
Passive: The antique vase was broken.
3. Use the passive when the agent or the actor is so unimportant or obvious that you do not need to mention it.
Rica was born in Seychelles.

32
4. Use a passive verb if you want to hide the name of the person who is responsible for an unpleasant decision or
result.
An increase in tuition fees was proposed.

When to use the passive presents the greatest challenge to ESL learners.

Forms of the English Passive


We usually form simple passives like these:
Paper is produced from trees. (simple present)
Paper was produced from trees. (simple past)

Here are other possible forms:


1. With modals
Paper can be produced from trees.
2. With present perfect
Paper has been produced from trees.
3. With present progressive
Paper is being produced from trees.
4. With past progressive
Paper were being produced from trees.
5. With be going to for future
Paper is going to be produced from trees.

PHRASAL VERBS
These are verbs which consist of two or three words. They consists of:
1. a verb followed by an adverb;
go up, spill over, push through
2. a verb followed by a preposition; or
come upon, reckon with, bank on
3. a verb followed by an adverb and a preposition
break out of, look forward to, go along with

Just like ordinary verb, phrasal verbs may be used:


1. intransitively
Why don’t you speak up?

2. transitively
Let’s cut pollution down to conserve our environment
3. both intransitively and transitively
A plane took off.
She took her coat off because it was warm.

Meaning
A two-word verb often has a one-word synonym, which is generally more formal. Here are some examples:
Phrasal Verb Synonym Phrasal Verb Synonym
call up telephone give in/up surrender
keep on continue leave out omit
pick out choose put off postpone

Separable and Inseparable Verbs


 Parts of inseparable phrasal verbs cannot be separated. If there is a direct object, it follows the phrasal verb.
Look after your baby brother.
Look after him
 On the other hand, the object of separable phrasal verbs is movable. A pronoun object comes between the first
and second part. A short noun object can come between the two parts or can follow the second part.
Donna turned on the light.
Donna turned it on.
 Some phrasal verbs can be either separable or inseparable according to their meanings in a certain context.
She passed out. (fainted)
She passed the brochures out. (distributed)

The car broke down. (stopped running)


The polite broke the door down. (opened by force)

Separable Phrasal Verbs with Their Objects


back it/the car up
blow it/the candle out
it/the balloon up
break them/the statistics down down
them/the negotiations off
bring it/the change about
it/the subject up
burn it/the building down
them/the papers up
clear them/the dishes away
it/the misunderstanding up
close it/the business down

33
draw it/the agreement up
fill it/a form in/out
it/the cup up
find it/the answer out
give it/this old bag way
it/eating candy up
it/the news out
hand it/the work in/out
keep them/expenses down
it/the radio on
leave it/the question out
let them/our friends in/out
lock them/the prisoners up
look them/our relatives in Manila up
make it/the handwriting out
it/a story up
mix it/food being prepared up
them/people up
pass it/the responsibility on
pay it/the money back
them/my enemies back
pick it/a new shirt out
point it/the problem out

ADJECTIVES
An adjective –
1. Is a word which describes or denotes the qualities of something
2. Commonly occurs between a determiner and a noun, or after be or other linking verbs or immediately following
the intensifier very
the _____ baby seems (very) _____
the hungry baby seems (very) hungry
3. Is associated with certain derivational morphemes
{-y} healthy, leafy
{-al} racial, normal
{-able} understandable, visible
{-ed} aged, learned
{-ful/-less} hopeful, hopeless
{-ish} childish, boyish
{-ive} active, native
{-ous} famous, marvellous
4. Has inflectional morphemes for comparative and superlative forms
pretty prettier prettiest
5. Modifies or complements nouns
the honest man (modifier)
The man is honest. (complement)
6. Has various types in terms of characteristic positions: ATTRIBUTIVE, which precede nouns, and PREDICATIVE,
which follow linking verbs
The diligent students pass the tough exam. (attributive)
They are happy with their high scores. (predicative)

Other related concepts


1. restrictive/nonrestrictive adjectives
RESTRICTIVE adjectives are necessary for defining which noun is being referred to while NON-RESTRICTIVE
adjectives merely add information without being essential for identification.
A concrete house (restrictive)
My uncle owns a house, built of concrete materials. (non-restrictive)
2. polarity
POLARITY refers to positive and negative contrasts in a language.
Positive polarity Negative polarity
big small, little
old young
long short
good bad
fast slow
Adjectives with positive polarity are unmarked forms because they are used more frequently in a given
language, learned earlier by children, and used in neutral contexts. Adjectives of negative polarity are marked.
They are less frequently used.
3. gradability
a. Adjectives can be placed in continuum of intensity, with the intensity increasing or decreasing depending
on the intensifier chosen.
[Less intense] [More intense]
Somewhat rare, rare, quite rare, very rare, extremely rare
 Adjectives that can be compared are also called gradable adjectives. Comparative forms (adjectives
marked by -er, more, or less) show differences/contrasts between two things or groups. Superlative
forms (marked by –est, most, or least) show differences in three or more things or groups.
 Comparison do nor apply to absolutes such as unique, possible, impossible, horizontal, round, square,
and fatal. They can co-occur with words like nearly and almost.
The accident was fatal.
34
The accident was nearly fatal.
The accident was almost fatal.

 The as . . . as construction is used to show that two things or groups are similar.
Ella is as tall as her mother.

Order of Adjectives in Noun Groups


When two or more adjectives are used in a structure, they usually occur in a particular order or sequence as
follows:
DET opinion size shape condition age color origin NOUN
many pretty small round chipped antique blue chinese vases

COORDINATION
Conjunction or coordination is the process of combining ideas. Two constituents of the same type can be put together to
produce another larger constituent of the same type. Traditional grammar calls this process compounding.
Compound sentence: The boys sang and the girls danced last night.
Compound subject: The teacher and her students will join the parade.
Compound verb: The children play and eat during recess.
Compound object: We boiled corn and cassava.

Conjoining like constituents as shown above is referred to as simple coordination. Here are other ways of coordinating
ideas:
1. Ellipsis: Omission or elision of the first verb phrase in the second and adding the word too or either (for
uninverted forms), and so or neither (for inverted forms).
Affirmative forms
My friends like to read storybooks and I, too. (uninverted)
A horse runs fast, and so does an ostrich. (inverted)
Negative forms
Donna can’t climb a tree, and his little brother can’t, either. (uninverted)
Ducks can’t fly high, and neither can chickens. (inverted)

2. Use of pro-form, i.e., the substitution of pronoun for a repeated noun.


Luis plays the guitar and he plays the harp, too.

3. Complex or correlative conjunctions like both . . . and..


My father is both kind and sincere.

Forms of Coordinating Conjunctions


Other than and, simple coordinating conjunctions include: for, nor, but, or, yet, and so. Note the following examples:
milk or chocolate
small but/yet terrible
The table is big, so heavy to lift. (non-clausal)
He came late, so he missed the fun. (clausal)
They accepted the verdict, for they failed to counter the charges against them. (clausal)

Other forms of correlative conjunctions are either . . . or, not only . . . but also, and neither . . . nor. These pairs are
used together
Either Tony or Nico will top the test.
Anna is neither friendly nor generous.
Our teacher is not only competent but also very understanding.

Use of Coordinating Conjunctions


Below is a straightforward account of the simple conjunctions:

Conjunction Meaning Conjunction Meaning


one or the other of two alternatives is
for because or
true
and plus yet but at the same time
conjoins two negative sentences,
nor so therefore
both of which are true
but shows contrast

A deeper and thorough study of each conjunction, however, reveals certain properties beyond the given straightforward
account. To illustrate, here are the other meaning and uses of and.
1. As logical operator (the truth-conditional meaning)
The entire conjoined statement is true so long as each conjunct that makes it up is true. If one conjunct is false,
then the statement is false.
2. As marker of many meanings
Celce-Murcia and Larsen-Freeman (2001) citing Posner (1980) provides these illustrations:
 Annie is in the kitchen, and she is making doughnuts. (and there . . .)
 Annie fell into a deep sleep, and her facial color returned. (and during this time . . .)
 The window was open, and there was a draft. (and coming from it . . .)
 Peter married Annie, and she had a baby. (and after that . . .)
 Paul pounded on the stone, and he shattered it. (and thereby . . .)
 Give me your picture, and I’ll give you mine. (If you give me your picture, I’ll give you mine.)
3. As inferential connective
A reader/listener can draw an inferential connection from sentences like Susan jumped and hurt her ankle. The
use of and invites the listener/reader to seek some other implicit relevant connection between stated conjuncts.
35
4. As marker of speaker continuation
In conversational discourse, sometimes a speaker uses and to signal that the utterance to follow is in some way
connected with what has come before. This particular use of and goes beyond the usual content conjunctive use;
rather it places and into the category of discourse markers like oh and well.

SUBORDINATION
SUBORDINATION means putting less important ideas in less important grammatical structures like dependent clauses.
One means of subordination is sentence combining or reducing.
Sentence combining
Melissa topped the test.
Melissa was late by twenty minutes.

Although late by twenty minutes, Melissa topped the test.
dependent clause independent clause

Although late, Melissa topped the test
dependent clause independent clause
Subordinating Conjunctions
Subordinating conjunctions do the job of connecting dependent clauses to independent clauses. Shown below are
different types.
Type Conjunction Type Conjunction
when, before, after,
time since, while, until, conditional if, unless
as
purpose in order to, so that reason because, since, as
although, though,
result so that concessive
while, despite
place where, wherever manner as, like

Time Her father died when she was young.


Conditional If I could afford it, I would buy a car.
Purpose They had to extend the session in order to discuss all concerns raised.
Reason I couldn’t ignore him because he was my childhood playmate
Result She had difficulty moving house so I extended help by transporting her things.
Concessive While I did well in class, I was a poor performer at club activities.
Place Wherever I stayed, I found troublesome neighbors.
Manner Is she often rude and cross like she’s been this past week?

Relative clauses
Another form of subordination involves the embedding of one clause within another. For example:
The lady came into the room.
The lady was small and slender.
 The lady [the lady came into the room] was small and slender.
The lady who came into the room was small and slender.

The most common relative pronouns which mark relative clauses are: that, which, who, whom, and whose. Their uses
are presented earlier in the section on pronouns

NEGATION
In English, negation affects words, phrases, and sentences.

Forms to Express Negation


The following forms mark negation in English (Celce-Mucia and Larsen-Freeman, 2001.):

Affix-Negation No-Negation Not-Negation


not, n’t
a- (atypical)
(I cannot/can’t) play the
dis- (dishonest)
piano.)
in/im/ir/il-
no (no plans) never (not + ever)
(inadequate/impossible/
nothing (My aunt has never left our
irrelevant/illegal)
nobody town.)
non- (non-formal)
no one neither (not + either)
un- (uncomfortable)
nowhere nor (and + not)
-less (useless)
Neither his brother nor his
-free (fat-free)
sister helps support him in
his studies.
Negation at the lexical or word level can simply use the negative affix. For example:
untidy untidily
impossible impossible
inadequate inadequately
illegally illegally
dishonest dishonestly
atypical atypically

Determining which affix to use is not always predictable. However, the choice of im-, in-, il- or ir- is phonologically
conditioned by the consonant which follows it, i.e., im- is used if the following consonant is bilabial (b, p, m), il- goes with
a stem beginning with l, and ir- with a stem beginning with r. The prefix in- is the most common.
Nothing, nobody, and no one are indefinite pronouns while nowhere is an adverb.
36
Other negative items include never (negative adverb of frequency), nor (negative coordinating conjunction, and
neither . . . nor (negative correlative conjunction.
The basketball players never admitted their mistake.
They preschoolers can neither read nor write, nor can they comprehend do mathematical computations yet.

At the phrase level, no can function as a negative determiner in a noun phrase.


No agreement has been reached yet.
No may also be followed by a gerund as in no reading, no parking, or no littering.
Not is used before infinitive verbs to make the phrase negative.
She reminded her friends not to forget their bathing suits.

At the sentence level, not or its contraction n’t is the main negator. This applies to different sentence types.
Mrs. Palma is not/isn’t our teacher. (statement)
Are you not/Aren’t we meeting today? (question)
Do not/Don’t laugh. (command)
Was it not/Wasn’t it exciting! (exclamation)

No and not are negative substitutes. No can be a negative substitute for a whole sentence while not for a subordinate
clause.
A: Is she coming with us?
B: No. She’ll do library work for an hour.
Are you joining us on Friday? If not, please let me know by tomorrow.
A: Is Pepito interested in the post?
B: I’m afraid not. He’d rather be a plain member.

Placement of not
1. Not usually follows the be-verb, whether functions as a main verb (copula) or an auxiliary/helping verb.
Surprisingly today, the birds are not noisy. (main)
I’m wondering why they are not chirping. (auxiliary verb)
2. Other than be, not follows the auxiliary verb if one is present or the first auxiliary (modal, phrasal modal, or have) if
there are two or more.
I cannot swim well.
The principal must not have been joking when he said that.
We have not been analyzing the data since we received them.
3. With other main verbs, a do-verb is introduced before negation can take place.
The child swims in the pool.  The child does swim in the pool.
The child does not swim in the pool.

YES/NO QUESTIONS

Inverted and Uninverted Yes/No Questions


YES/NO QUESTIONS are often defined as questions for which either “yes” or “no” is the expected answer. They are
produced with a rising intonation.
Yes/no questions are formed by inverting the subject and the operator.
Lucy is your cousin.  Is Lucy your cóusin?
She can speak fluently.  Can she speak flúently?
She has been a consistent debater.  Has she been a consistent debáter?
She loves (= does love) to read opinion columns.  Does she love to read opínion columns?

Yes/no questions may have a statement word order, i.e., the word order is uninverted. This sentence, however, is
likewise said with a rising intonation.
2
Lucy is your 3cousin3↑
2
She can speak 3fluently3↑

Answers to Yes/No Questions


Yes/no questions usually take short answers using the operator. The operator is italicized below.
1. Is your sister fond of sweets? {Yes, she is.
{No, she isn’t.
{*Yes,she’s.

2. Can you speak Chinese? {Yes, I can.


{No, I can’t.

3. Are we supposed to attend? {Yes, we are.


{No, we aren’t

4. Have they eaten? {Yes, they have.


{No, they haven’t

5. Does the baby walk? {Yes, it does.


{No, it doesn’t.

If the sentence contains more than one auxiliary verb, the short answer may also contain an auxiliary verb in addition to
the operator.
Will they have joined? {Yes, they will have.
{No, they won’t have.

If the second or third auxiliary is a be form, it is usually omitted.


37
Will she be able to pass? {Yes, she will.
{No, she won’t.
Negative Yes/No Questions
Semantic problems may arise for many ESL learners who react to a negative yes/no question in a literal manner in their
language. This means that they agree or disagree with the form of the yes/no question, thus causing miscommunication.
Don’t you feel sorry? {Yes (I don’t feel sorry).
{No (I feel sorry).

Among native speakers of English, the expected response is:


Don’t you feel sorry? {Yes, (I feel sorry).
{No, I don’t feel sorry}.

Focused Yes/No Questions


While neutral yes/no questions, as in the preceding cases, query on the whole state, activity or event, this query can be
more focused sometimes.

Does Álex plan a foreign trip with Melly? (or did someone else?)
Does Alex plán a foreign trip with Melly? (or did he only suggest?)
Does Alex plan a foreign tríp with Melly? (or is it something else?)
Does Alex plan a foreign trip with Mélly? (or is it with someone else?)

The focused sentence element gets the primary stress as shown above.

Some Versus Any in Questions


Some and any can both occur with different question types depending on the meaning.
 In open or unmarked questions, any is used in questions as well as in negatives.
Is there any sugar?
There isn’t any sugar.

 However, some is used in questions that expect a positive response, e.g., an offer:
Would you like some cold drink? (encourages a “yes” answer)

 Here are questions to consider:


Is there some relief? (expects a “yes” answer)
Is there any relief? (neutral question/no special meaning involved)
Isn’t there some relief? (Surely there is.)
Isn’t there any relief? {
{(hopeful that there would be)
Is there no relief? {

Other Functions of Yes/No Questions


1. Direct request: Can I borrow your notes on phonology?
2. Less direct request: Could I borrow your notes on phonology?
3. Polite request: I wonder if I could borrow your notes on phonology.
4. Offers or invitations: Would you like to have a cup of coffee?
5. Commands Would you please pay attention?
6. Reprimands Don’t you have enough sense to do such a thing?
7. Complaints Have you ever tried using this gadget at all?

WH-QUESTIONS
WH-QUESTIONS are used to seek specific information so they are also referred to as information questions. Except for
how, these words begin with wh- : who, whose, whom, what, which, where, when, why, and how.

A variety of constituents can be queried in a wh-question. Consider this sentence:


Liza bought a beautiful house for her parents before she went to Canada.

Subject NP: Who bought a beautiful house? (Liza)


Object NP: What did Liza buy? (a beautiful house)
Object of the For whom did Liza buy a beautiful house? (her parents)
preposition: Who(m) did Liza buy a beautiful house for? (her parents)
Verb phrase: What did Liza do when she came home? (She bought a beautiful house).
Determiner: Whose parents did Liza buy a beautiful house for when she came home? (her parents)
Adjective: What kind of house did Liza buy? (a beautiful house)
Adverbial: When did Liza buy a beautiful house? (before she went to Canada)
Adverbial: Where did Liza go? (to Canada)

Wh-questions elicit specific kinds of information.


What? The answer is non-human.
Who? The answer will be human.
Which? The answer is one of a limited group.
When? The answer will be a time or an occasion.
Where? The answer will be a place or situation.
Why? The answer will be reason.
How? The answer will show manner, means, or degree.
How much? The answer will be connected with an uncountable noun.
How many? The answer will be connected with a countable noun.
How often? The answer will indicate frequency.

Forming Wh-Questions
38
If who, what, or which is the subject of the sentence, it is followed by the normal word order of a statement.

Statement: Grammar study is exciting.


Question: What is exciting?

Statement: Those big dogs chased the cat.


Question: Which dogs chased the cat.

Statement: Their teacher gave a test.


Question: Who gave a test?

Whom/who, what and which as objects form questions by putting the wh-words first, and do, does, or did next.
Statement: He planted fruit trees.
Question: What did he plant?
Statement: Mothers bathes my baby sister.
Question: Who (Whom) does my mother bathe?

Statement: The children catch yellow butterflies.


Question: Which butterflies do the children catch?

A modal (e.g., can) cannot be replaced by do, does, or did. The do-verb replaces the main verb.
Statement: My three-year-old sister can read.
Question: What can my sister do?

Social Uses of Wh-Questions


Certain fixed formulaic wh-questions serve social functions (Celce-Murcia and Larsen Freeman 2001). Among them are:
Introductions: How do you do?
Greetings: How are you? How have you been? What’s up? What’s new?
Eliciting personal reactions: How was the test?
Making suggestions: Why don’t you seek advice? How about a trip?
Responding positively Why not?
to a suggestion:
Expressing exasperation: What now?
Seeking another’s opinion: How about you? What do you think?
Challenging another’s opinion: What for? How come? Since when?
Expressing perplexity: What to do?
Asking for clarification/expansion: What about it?

ADVERBS
Adverbs modify or change the meaning of other words such as verbs, adjectives, another adverb, or even a whole
sentence.
The athlete can run fast. (verb modifier)
Sailboats are really beautiful to watch. (adjective modifier)
The athlete can run very fast. (adverb modifier).
Perhaps, Nena’s family will give a party.(sentence modifier)

Adverbs or adverbials vary in form as follows:


Adverbial clause: The child cried because he was hungry.
Adverbial phrase: Diane sang very sweetly.
Prepositional phrase: She sang during our class reunion.
Word: We eagerly look forward to your graduation.

Adverbs can be readily recognized through certain affixes. For example:


1. Suffix –ly hopefully, popularly
2. Prefix a- aloud, adrift, anew
3. Suffix –wise lengthwise, clockwise
4. Suffix –wards backward(s), forward(s)

Kinds of adverbs
1. Adverbs of frequency: answer the question how often?
(always, never, usually, rarely)

2. Adverbs of relative time can be used with all tenses as meaning permits
(just, still, already, lately)

3. Adverbs of manner answer the question how? or how well?


(carefully, eagerly, clearly, quickly)

4. Adverbs of place answer the question where?


(here, in the city)

5. Adverbs of time answer the question when?


(today, on April 15)

7. adverbs that emphasize only and even

Where we put only makes a big change in the meaning of a clause. To illustrate:
1. Only he (no one else) invited Alex to join the team this year.
2. He only invited Alex to join the team this year this year. (not ordered)
39
3. He invited only Alex (no one but Alex) to join the team this year.
2. He invited Alex only to join the team this year. (to join, not to do anything else)
3. He invited Alex to join the team only this year. (Before an adverb of time, only means as recently as or at no
other time.)

Positions of Adverbials
While some adverbials are fixed in their positions in the sentence, others are movable. They can occur sentence initially,
medially, or finally.
Sentence-initial: Doubtlessly, we must conclude that the findings are correct.
Sentence-medial: We, doubtlessly, must conclude that the findings are correct.
Sentence-final: We must conclude that the findings are correct, doubtlessly.

Order of Adverbials
When two or more adverbials co-occur in final position in the same sentence, ordering should be observed.

{direction} + position ↔ manner + time ↔ frequency + {purpose}


{goal } {reason}
He walks home leisurely every day because he wants to feel relaxed.

40

You might also like